You are on page 1of 63

Extra Edge Assignment

Subject : Mathematics

Assignment 1

1.

ABC is triangle. Circles C1 , C2 and C3 are drawn tieh sides AB, BC and CA as their diameters. The radical
axis between any two circles w.r.t. the ABC is one of its
(A) angle bisector
(B*) altitude
(C) median
(D) perpendicular bisector of the sides

2.

The function f(x) defined on the real numbers has the property that f(f(x)) . (1 + f(x)) = f(x) for all x in the
domain of f. If the number 3 is the domain and range of f, then the value of f(3) equals
(A) 3/2
(B*) 3/4
(C) 1/4
(D) 1/2

3.

If m and b are real numbers and mb > 0, then the line whose equation is y = mx + b cannot contain the point
(A) (0, 2006)
(B*) (2006, 0)
(C) 0, 2006)
(D) (19, 97)

4.

Number of seven digit whole numbers in which only 2 and 3 are present as digits if no two 2s are consecutive
in any number, is
(A) 26
(B) 33
(C) 32
(D) 53

5.

If f(x) = x4 + ax3 + bx2 + cx + d be polynomial with real coefficient and real roots. If |f(i)| = 1, where i 1 ,
then a + b + c + d is equal to
(A) 1
(B) 1
(C) 0
(D*) can not be determined

6.

Let ABCDE is a regular pentagon with all sides equal to 4. Which one of the following is a correct solution for
the length AC ?
(i)
2 cos (18)
(ii)
2 sec (72)

32 32 cos(108o )

(iii)

(A) only (i) and (ii) are correct


(C) only (iii) and (i) are correct

(B*) only (ii) and (iii) are correct


(D) all are correct

7.

A line x = k intersects the graph of y = log5x and the graph of y = log5(x + 4). The distance between the points

8.

The radius of the circle passing through the vertices of the triangle ABC, is

of intersection is 0.5. Given k = a b , where a and b are integers, the value of (a + b) is


(A) 5
(B*) 6
(C) 7
(D) 10

(A)

8 15
5

(B*)

(C) 3 5
9.

3 15
5

(D) 3 2

The area of the region consisting of all points (x, y) so that x 2 + y2 1 |x| + |y| is
(A)
(B) 1
(C*) 2
(D) 3

10.

21

13

7
23

3
15

9
25

5
17

11
27

19

29

A
12
B

12
6

Consecutive odd numbers are arranged in rows as shown. If the rows are continued in the same pattern then
the middle number of row 51, is
(A*) 2601
(B) 2500
(C) 2704
(D) 2401
11.

The expression [x + (x3 1)1/2]5 + [x (x3 1)1/2]5 is a polynomial of degree


(A) 5
(B) 6
(C*) 7
(D) 8

Extra Edge Assignment

Subject : Mathematics

Assignment 2

1.

Let a, b, c, d, e, f, g, h be distinct elements in the set {7, 5, 3, 2, 2, 4, 6, 13}. The minimum possible
value of (a + b + c + d)2 + (e + f + g + h)2 is
(A) 30
(B) 32
(C) 34
(D) 40

2.

If the circles
and

x2 + y2 + 2ax + 2by + c = 0
x2 + y2 + 2bx + 2ay + c = 0

where c > 0, have exactly one point in common then the value of
(A) 1
3.

(C) 2

(D) 1/2

Given ABC is inscribed in the semicircle with diameter AB. The area of ABC equals 2/9 of the area of the
semicircle. If the measure of the smallest angle in ABC is x then sin 2x is equal to
(A)

4.

(B)

(a b ) 2
is
2c

The value of
(A) 1

i
100

n0

2
9

(B)
n!

equals (where i =
(B) i

18

(C)
1 )

(D)

(C) 96 + i

(D) 97 + i

5.

The value of b > 0 for which the region bounded by both the x-axis and y = |2x| + b has an area of 72, is

6.

If 500! = 2m.N, where N is an odd positive integer, then m is equal to


(A) 452
(B*) 494
(C) 498

7.

Let f be a linear function for which f(6) f(2) = 12. The value of f(12) f(2) is equal to
(A) 12
(B) 18
(C) 24
(D*) 30

8.

(A*) 12

(B) 36

2 1

1
= tan1
tan
2

(A) is equal to 1

(C) 6 2

1
1
2 and = tan (3) sin

(B) is equal to 0

(D) 144

(D) none of these

5 , then cot ( )

(C) is equal to

2 1

(D) is non existent

9.

There are three teachers and six students. Number of ways in which they can be seated in a line so that
between any two teachers there are exactly 2 students, is
(A) 3 . 3! . 6!
(B) 2 . 6!
(C) 2 . 3! . 6!
(D) 3 . 6!

10.

The average of the numbers n sin n for n = 2, 4, 6,....., 180


(A) 1

11.

12.

(B) cot 1

(C) tan 1

A circle with centre O is tangent to the coordinate axes


and to the hypotenuse of the 30 60 90 triangle ABC
as shown, where AB = 1. To the nearest hundredth, the
radius of the circle is
(A) 2.37
(B) 2.24
(C) 2.18
(D) 2.41
If s = 1 +
(A)

s
2

(B)

+ ......, then 1

1
s
4

(C)

(D)

1
2

y
O

42

s
2
2

60
A 1 B

+........ equals
(D)

s 1
2

Extra Edge Assignment

Subject : Mathematics
1.

x1 / x
1

Find the value of x that satisfying the equation log 1/( x 1) =


5050
x

(A) 1

2.

Assignment 3

(B) 10

(C) 100

(D) 1000

The set of points (x, y) whose distance from the line y = 2x + 2 is the same as the distance from (2, 0) is a
parabola . This parabola is congruent to the parabola in standard form y = Kx 2 for some K which is equal to
(A)

5
12

5
4

(B)

(C)

(D)

12

3.

The number 2006 is made up of exactly two zeros and two other digits whose sum is 8. The number of 4 digit
numbers with these properties (including 2006) is
(A) 7
(B) 18
(C) 21
(D) 24

4.

Let f be the function defined by f(x, y, z) =

5.

The smallest possible value of f, is


(A) 9
(B) 8

( x y z )( xy xz yz )
for all positive real numbers x, y and z.
xyz
(C) 6

The right-angled triangle has two circles touching its sides


as shown. If the angle at R is 60 and the radius of the
smaller circle is 1, then the radius of the larger circle is
(A) 2 3

(B) 2

(C) 2 2

(D) 3

(D) 3

6.

Let f be a function defined from R+ R+ . If [f(xy)]2 = x(f(y))2 for all positive numbers x and y and f(2) = 6 then
f(50) is equal to
(A) 10
(B) 30
(C) 40
(D) 50

7.

An equilateral triangle, with sides of 10 inches, is inscribed in a square ABCD in such a way that one vertex
is at A, another vertex on BC and one on CD. The area of the square is

8.

(A) 25 (2 3 )

(B) 25 (2 3 )

(C) 25

The coefficient of x3 in the expansion of (1 + x + x2)12, is


(A) 352
(B) 350
(C) 342

(D)

100

2 3

(D) 332

9.

The radius of the inscribed circle and the radii of the three escribed circles of a triangle are consecutive
terms of a geometric progression then triangle
(A) is acute angled
(B) is obtuse angled
(C) is right angled
(D) is not possible

10.

A function f is defined for all positive integers and satisfies f(1) = 2005
and f(1) + f(2) + ...... + f(n) = n2 f(n)
for all n > 1. The value of f(2004) is
(A)

11.

12.

1
1002

(B)

1
2004

(C)

2004
2005

(D) 2004

The line (k + 1)2 x + ky 2k2 2 = 0 passes through a point regardless of the value k. Which of the following
is the line with slope 2 passing through the point ?
(A) y = 2x 8
(B) y = 2x 5
(C) y = 2x 4
(D) y = 2x + 8

If the solution set for f(x) < 3 is (0, ) and the solution set for f(x) > 2 is ( , 5), then the true solution set
for (f(x))2 f(x) + 6, is
(A) ( , )
(B) ( , 0]
(C) [0, 5]
(D) ( , 0] [5, )

Extra Edge Assignment

Subject : Mathematics

1.

Assignment 4

ABCD is a quadrilateral with an area of 1 and BCD = 100, ADB = 20,


AD = BD and BC = DC shown in figure. The product (AC) (BD) is equal to

2
(A)
3

2 3
(B)
3

(C)

4 3
(D)
3

20

100 C

2.

Locus of the feet of the perpendicular from the origin on a variable line passing through a fixed point (a, b)
(where a 0, b 0) is a circle with x-intercept p and y-intercept q, then
(A) p = 0 and q = 0
(B) p = 0 and q 0
(C) p 0 and q = 0
(D) p 0 and q 0

3.

Two rods AB and CD of length 2a and 2b respectively (a > b) slides on the x and y axies respectively such
that the points A, B, C and D are concyclic. The locus of the centre of the circle through A, B, C and D is a
conic whose length of the latus rectum is
(A)

2b 2
a

(B) 2a a 2 b 2

(C) 2ab a 2 b 2

(D) 2 a 2 b 2

x is rational
1 if
Let f(x) =
x is irrationa
0 if
A function g(x) which satisfies x f(x) g(x) for all x is
(A) g(x) = sinx
(B) g(x) = x
(C) g(x) = x2

(D) g(x) = |x|

5.

How many of the 900 three digit number have atleast one even digit ?
(A) 775
(B) 875
(C) 450

(D) 750

6.

cot 10 + tan 5 equal to


(A) sec 10
(B) sec 5

(D) cosec 10

7.

A sequence of equilateral triangles is drawn. The altitude of each is

8.

Two circles with centres at A and B, touch at T. BD is the tangent


at D and TC is a common tangent. AT has length 3 and BT has
length 2. The length CD is
(A) 4/3
(B) 3/2
(C) 5/3
(D) 7/4

4.

(C) cosec 5

3 times the altitude of the preceding

triangle, the difference between the area of the first triangle and the sixth triangle is 968 3 square unit. The
perimeter of the first triangle is
(A) 10
(B) 12
(C) 16
(D) 18
B

T
C

9.

The value of cos 5 + cos 77 + cos 149 + cos 221 + cos 293 is equal to
(A) 0
(B) 1
(C) 1
(D) 1/2

10.

Let C be the circle of radius unity centred at the origin. If two positive numbers x1 and x2 are such that the line
passing through (x1, 1) and (x2, 1) is tangent to C then
(A) x1 x2 =1
(B) x1x2 = 1
(C) x1 + x2 = 1
(D) 4x1 x2 = 1

11.

12.

Suppose that and z are complex numbes such that both (1 + 2i) and (1 + 2i) z are different real numbers.
The slope of the line connecting and z in the complex plane is
(A) 2
(B) 1/2
(C) 2
(D) can not be determined
x sec y tan 2 cos
If
then y equals
tan y sec cot

(A)

cos 2
sin

(B) sin

(C) cos

(D) sin2

Extra Edge Assignment

Subject : Mathematics

Assignment 5

1.

the locus of the point of intersection of the tangent to the circle x 2 + y2 = a2, which include an angle of 45 is
the curve (x2 + y2)2 = a2 (x2 + y2 a2). The value of is
(A) 2
(B) 4
(C) 8
(D) 6

2.

Consider the circle x2 + y2 14x 4y + 49 = 0. Let 1 and 2 be lines through the origin O that are tangent
to the circle at points A and B. If the measure of angle AOB is tan1() then equal to
(A)

3.

4.

2
7

(B)

21
45

(C)

28
45

(D) none

1
1
1
1

1 1
tan 1 tan 1
tan 1
tan 1 is an integer which is
The value of expression, 14 tan tan
3
7
13
21
31

equal to
(A) 2
(B) 5
(C) 7
(D) 10

If a, b are positive real numbers such that a b = 2, then the smallest value of the constant L for which
x 2 ax

(A) 1/2

x 2 bx < L for all x > 0, is

(B) 1/ 2

(C) 1

(D) 2

5.

If every solution of the equation 3 cos2x cosx 1 = 0 is a solution of the equation a cos22x + b cos 2x 1
= 0. Then the value of (a + b) is equal to
(A) 5
(B) 9
(C) 13
(D) 14

6.

What is the y-intercept of the line that is parallel to y = 3x, and which bisects the area of a rectangle with
corners at (0, 0), (4, 0), (4, 2) and (0, 2) ?
(A) (0, 7)
(B) (0, 6)
(C) (0, 5)
(D) (0, 4)

7.

Let f(x) = x2 + kx ; k is a real number. The set of values of k for which the equation f(x) = 0 and f(f(x)) = 0 have
exactly the same real solution set is
(A) (0, 4)
(B) [0, 4)
(C) (0, 4]
(D) [0, 4]

8.

If x log3 ( 4 ) 27 , then the vlaue of x (log3 4 )


(A) 4
(B) 16

9.

If Q is the point on the circle x2 + y2 10x + 6y + 29 = 0 which is farthest from the point P(1, 6), then the
distance from P to Q is

10.

(A) 2 5

(D) 81

(B) 2 7

(C) 4 5

(D) 4 7

triangle is sq. units is

(B) 12 7 3

(C) 12 + 7 3

(D) 4

3 . Then the area of

If 0 < < 2 , then the intervals of values of for which 2 sin2 5 sin + 2 > 0, is

(A) 0 , , 2
6

(B) ,
8 6

(C) 0 , ,
8

6 6

12.

(C) 64

Given an isosceles triangle, whose one angle is 120 and radius of its incircle is
(A) 7 + 12 3

11.

41
,
(D)
48

If w = + i where 0 and z 1, satisfies the condition that


of z is

(A) {z : |z| = 1}

(B) {z : z z )

w wz
is purely real, then the set of values
1 z

(C) {z : z 1}

(D) {z : |z| = 1, z 1}

Extra Edge Assignment

Subject : Mathematics

1.

2.

3.
4.

Assignment 6

Let a, b, c be the sides of triangle. No two of them are equal and R. If the roots of the equation
x2 + 2(a + b + c)x + 3 (ab + bc + ca) = 0 are real, then
1 5
4 5
(C) ,
(D) ,
3 3
3 3
If r, s, t are prime numbers and p, q are positive integers such that the LCM of p, q is r2 t4s2, then the numbers
of ordered pair of (p, q) is
(A) 252
(B) 254
(C) 225
(D) 224

(A) <

4
3

(B) >

5
3


Let 0 , and t1 = (tan )tan , t2 = (tan )cot , t3 = (cot )tan , t4 = (cot )cot, then
4
(A) t1 < t2 < t3 < t4
(B) t4 < t3 < t1, t2
(C) t3 < t1 < t2 < t4
(D) t2 < t3 < t1 < t4

Internal bisector of A of a triangle ABC meets side BC at D. A line drawn through D perpendicular to AD
intersects the side AC at E and the side AB at F. If a, b, c represent sides of ABC then
(A) AE is harmonic mean of b and c
(C) EF =

4bc
A
sin
bc
2

(B) AD =

2bc
A
cos
bc
2

(D) the triangle AEF is isosceles


PA 2 PB 2 PC 2 PD 2

5.

If P is a point on C1 and Q in another point on C2, then

6.

A circle touches the line L and the circle C1 externally sucht that both the circles are on the same side of the
line, then the locus centre of the circle is
(A) ellipse
(B) hyperbola
(C) parabola
(D) parts of straight line

7.

A line M through A is drawn parallel to BD. Pointo S moves such that its distance from the line BD and the
vertex A are equal. If locus of S cuts M at T 2 and T3 and AC at T1, then area of T1 T2T3 is
(A) 1/2 sq. units
(B) 2/3 sq. units
(C) 1 sq. units
(D) 2 sq. units

8.

If the roots of the equation x2 10cx 11d = 0 are a, b and those of x2 10ax 11b = 0, then find the value
of a + b + c + d. (a, b, c and d are distinct numbers)

(A) 3/4

(B) 3/2

QA 2 QB 2 QC 2 QD 2
(C) 1
(D) 9

is equal to

9.

3 3
3
3
If An = + + ....... + (1)n 1 and Bn = 1 An, then find the minimum natural number
4 4
4
4

10.

sin120 o
The value of
o
o
o
o
16 cos 15 . cos 30 . cos 120 cos 240

n0 such that Bn > An . n > n0

(A)

2 3
8

(B)

3 1
4

is

(C)

2 3
4

(D) 2 3

11.

Let S denote the set of numbers m such that the line y = mx does not intersect the parabola y = x 2 + 1. S
is a bounded interval. The length of S is
(A) 3
(B) 3.5
(C) 4
(D) 4.5

12.

A line 1 has a slope of (2) and passes through the point (r, 3). A second line 2, is perpendicular to 2 ,
intersects 1 at (a, b), and passes through the point (6, r). The value of 'a' is equal to
(A) r

(B)

2r
5

(C) 2r 3

(D)

5r
2

Extra Edge Assignment

Subject : Mathematics

Assignment 7

1.

711 + 1511 when divided by 22 leaves the remainder :


(A) 0
(B) 1
(C) 7

2.

The coefficient of x in the expansion of (1 + x) (1 + 2x) (1 + 3x)........(1 + 100x), is


(A) 4950
(B) 5000
(C) 5050
(D) 5100

3.

Suppose ABC is an equilateral triangle and P is a point interior to ABC. If the distance from P to sides AB,
BC and AC is 6, 7 and 8 units repsectively, then the area of the ABC , is
(A) 147 3

4.

(B)

147 3
2

(C)

21 3
2

(D)

441
3

If the graphs of y = cosx and y = tanx intersect at some value say in the first quadrant. Then the value of
sin is
(A)

5.

(D) 10

1 2
2

If S = 1 +
(A)

S
2

(B)

1 3
2

(C)

1 5
2

3S
4

(C) S

(D)

1
1
1
1
1
1
+
+
+ ......, then 1 +
+
+
+ ......... equals
9
16
9
25
49
4

(B)

1
4

1 5
2

(D) S

1
2

6.

How many solutions are there for the equation cos 2x sin22x = 0 on [0, 2] ?
(A) 6
(B) 4
(C) 2
(D) 1

7.

Number of ways in which 7 people can be divided into two teams, each team having at least one member, is
(A) 72
(B) 32
(C) 144
(D) 63

8.

Let P be a point on the complex plane denoting the complex number z. If (z 2) ( z i) is a real number then
the locus of P is
(A) y = 2x + 1
(B) 2y = 2 x
(C) y = x 2
(D) y = 2x 1

9.

The positive value of x that satisfies


(A)

10.
11.

1
n ( 4 15 )
2

(B)

10 = ex + ex, is

1
n ( 4 15 )
2

(C)

1
n ( 4 17 )
2

(D)

1
n ( 4 17 )
2

1
Let f : R{0} R be any function such that f(x) + 2f = 3x. The sum of the values of x for which f(x) =1 is.
x
(A) 1
(B) 2
(C) 1
(D) 2

Let r1, r2, r3 be the roots of the equation x4 9x3 + ax2 + bx + 16 = 0

where a, b are constants. Then the difference between the arithmetic mean
geometric mean
12.

13.

(A) 5/2

r1r2r3r4 of the roots, is equal to

(B) 1/2

(C) 17/4

(D) 1/4

r1 r2 r3 r4
and the
4

Sum of the x and y intercept of the circle described on the line segment joining (2, 1) and (1, 2) as diameter
is
(A) 1
(B) 2
(C) 3
(D) 4
If sin16a =
(A) 4

1
1
2
4
1
, then the value of
+
+
+
is
2
2
4
5
cos a
1 sin a
1 sin a
1 sin8 a
(B) 6
(C) 8
(D) 10

Extra Edge Assignment

Subject : Mathematics
Assignment 8
1.
A variable circle touches the x-axis and also touches the circle with centre at (0, 3) and radius 2. The locus
of the centre of the variable circle is
(A) an ellipse
(B) a circle
(C) a hyperbola
(D) a parabola

1
1
>
is
x 1
x2
(C) x < 2 and x < 1

2.

The set of real numbers x satisfying

3.

A sequence of three real numbers forms an arithmetic progression with a first term of 9. If 2 is added to the
second term and 20 is added to the third term, the three resulting number form a geometric progression. The
smallest possible value for the third term of the geometric progression, is
(A) 1
(B) 3
(C) 4
(D) 6

4.

The curve such that each point P on the curve has equal distances from the point (2, 2) and from the line
y = x has the equation
(A) (x + y)2 = 4
(B) (x y)2 = 4
2
(C) (x + y) + 8(y x) = 16
(D) (x + y)2 + 2(x + y) = 16

5.

for any straight line, let m and b represent its slope and y-intercept, respectively. Consider all lines having the
property that 2m + b = 3. These lines all have the specific point (x1, y1) in common. The ordered pair,(x1, y1)
is equal to
(A) (2, 3)
(B) (3, 2)
(C) (1, 2)
(D) (2,1)

6.

If the sum of the solutions of the equation sin2x sinx = cos2x


on the interval [0, 2] is expressed as a/b, where a and b are positive integers, a/b in lowest terms then
(a + b) is
(A) 8
(B) 9
(C) 10
(D) 11

7.

Quadratic equation with real coefficients whose one root is (2 + i) (3 i) wree i =

8.

Which of the following is equal to sec(t) + tan(t) ?

(A) x > 2

(B) x < 1

(A) x2 14x + 48 = 0
(C) x2 + 14x 48 = 0
t
(A) cot
2 4

(D) 1 < x < 2

(B) x2 14x + 50 = 0
(D) x2 + 14x + 49 = 0

t
(B) cot
2 4

(C)

1
cot t
2

(D)

1 , is

1
cot t
2
2

9.

If f(x) = x + 4 and g(f(x)) = 2x + 1, then the function g(x) is


(A) 2x 7
(B) 8x + 3
(C) 2x + 9

10.

In a triangle ABC, A = 72, b = 2 and c =

11.

Let T = {1, 2, 3, 4, 5}. A function f : T T is said to be one-to-one if t1 t2 implies that f(t1) f(t2). Obtain a oneto-one function such that t + f(t) is a perfect square for every t in T.

12.

(A) obtuse isosceles

(D) 2x2 + 5x + 4

5 1 , then the triangle ABC is


(B) acute isosceles
(C) right isoscels
(D) not isosceles

ab 1
bc 1
ca 1
+ cot1
+ cot1

If a > b > c > 0 then find the value of cot1


ab
bc
c a

13.

Find the equation to the locus of the centre of all circles which touch the line x = 2a and cut the circle x 2 +
y2 = a2 orthogonally.

14.

Let f(x) =

( x 4)( x 2 4x 5)

( x 2 2x 3)( 4 x 2 )

. Find

(A) the domain of f(x)

(C) all x such that f(x) >0

(B) the roots of f(x)

(D) all x such that f(x) < 0

Extra Edge Assignment

Subject : Mathematics

Assignment 9

1.

The points (6, 1), (6, 10), (9, 6) and (3, 3) are the vertices of a rectangle. What is the area of the portion
of this rectangle that lies above the x axis ?

2.

Let f(x) =

3.

Two circles of different radii R and r touch each other externally. The three common tangents form a triangle.

ax 2 bx . Find the set of real values of 'a' for which there is at least one positive real value of 'b'
for which the domain of f and the range of f are the same set.

Show that the area of the triangle is

2(Rr )3 / 2
Rr

4.

A variable straight line whose length is C moves in such a way that one of its end lies on the x-axis and the
other on the y-axis. Show that the locus of the feet of the perpendicular from origin on the variable line has the
equation, (x2 + y2)3 = C2x2y2.

5.

Evaluate : Lim
x a

6.

Let t1, t2 and t3 be the length of the tangents drawn from a point (x 1, y1) to the circles x2 + y2 = a2, x2 + y2 = 2ax
and x2 + y2 = 2ay respectively. The lengths satisfy the equation t14 = t22 t32 + a4. Show that locus of (x1, y1)
consists of x + y = 0 and x2 + y2 = a(x + y)

7.

Let an = 2 cos

8.

Consider a function f : x

2 cos 1
1, then show that Lim
(a1 a2a3.....an 1an) =
,R
n
3

the range of f

xa
; x R {1} where a is a real constant. If f is not a constant function, find
x 1
(ii)

f 1, is it exist

(iii)

f f f 1
f

f
(
f
(
x
))

f (x)
Given Lim
= 2, then evaluate the following limits, giving explicit reasoning
x 0 x 2

(i)

10.

n(e x e a )

the following.
(i)

9.

cos x n ( x a)

Lim [f(x)]
x 0

f (x)
(ii) Lim
where [x] denotes greater integer function
x 0
x

2 1
Find the sum to n term of the series Sn = cot1 2 + cot1
2

terms. Also deduce that Lim


Sn = cot1 2
n

1
3
2 2 + cot1
2

1
4
2 3 + ...... upto n
2

11.

The vertices of a triangle are A(x1, x1 tan 1), B(x2, x2 and 2) and C(x3, x3 tan 3). If the circumcentre O of the

12.

If (1 + sin t)(1 + cot t) =

13.

10 identical ball s are distributed in 5 different boxes kept in a row and labled A, B, C, D and E. Find the
number of ways in which the balls can be distributed in the boxed if no two adjacent boxes remain empty.

cos 1 cos 2 cos 3


x
triangle ABC is at the origin and H ( x , y ) be its orthocentre, then show that y = sin sin sin
1
2
3
5
. Find the value (1 sin t) (1 cos t) .
4

Extra Edge Assignment

Subject : Mathematics
1.

2.

Assignment 10

Tangent are drawn from any point on the circle x2 + y2 = R2 to the circle x2 + y2 = r2. Show that if the line joining
the points of intersection of these tangents with the first circle also touches the second , then R 2r

n(2 cos 2x )
2
n (1 sin 3 x )
Let a function f(x) be defined as f(x) =
e sin 2 x 1
n(1 tan 9 x )

for x 0
for x 0

Find the whether it is possible to define f(0) so that 'f' may be continuous at x = 0.
3.

4.

Find all possible values of a and b so that f(x) is continuous for all x R if

| ax 3 |,

| 3 x a |,
b sin 2x
2b
f(x) =
x

2
cos x 3

if

x 1

if

1 x 0

if

if

0x

Prove that, in a ABC, the median through A divides the angle A into two parts whose cotangents are,
2 cotA + cot C and 2 cot A + cot B and it makes an angle with the side BC whose cotangent is

1
(cot B cot C).
2

5.
6.

6 1
2

cos 1
Find the value of y = sin cot1 cos tan1 where x = cosec cos 1
2 3

ax sin x
If 'a' is a fixed constant then Lim
is equal to
x 0
x3

(A)
7.
8.

a2
6

(B)

a3
3

(C)

a2
3

a3
6

If f(x) = x2 + bx + c and f(2 + t) = f(2 t) for all real numbers t, then which of the following is true ?
(A) f(1) < f(2) < f(4)
(B) f(2) < f(1) < f(4)
(C) f(2) < f(4) < f(1)
(D )f(4) < f(2) < f(1)

Let P > 0 and suppose ABC is an isosceles right triangle with area P sq. units. The radius of the circle that
passes through the points A, B and C, is
(A)

(B)

2P

(C) 2 P

9.

Number of real solutions of the equation cosx + cos ( 2 x ) = 2, is


(A) 0
(B) 1
(C) 2

10.

A quadratic polynomial y = ax2 + bx + c has its vertex at (4, 5)


and has two x-intercept, one positive and one negative as shown.
Which one of the following must be negative ?
(A) only a
(B) only b
(C) only c
(D) only b and c

11.

(D)

P
2

(D)

(D) infinite
O
B

If in a ABC, the altitudes from the vertices A, B, C on opposite sides are in H.P., then sinA, sinB, sinC are
in
(A) G.P.
(B) A.P.
(C) A.G.P.
(D) H.P.

Extra Edge Assignment

Subject : Mathematics

1.

2.

3.

Assignment 11

Suppose that two circles C1 and C2 in a plane have no points in common. Then
(A) there is no line tangent to both C1 and C2.
(B) there are exactly four lines tangent to both C1 and C2.
(C) there are no lines tangent to both C1 and C2 or there are exactly two lines tangent to both C1 and C2.
(D) there are no lines tangent to both C1 and C2 or there are exactly four lines tangent to both C1 and C1 .

Number of three term arithmetic progressions which exist in the set {1, 2, 3, ........., 40} and common
difference d 0, is
(A) 190
(B) 200
(C) 380
(D) 400

x
1

The smallest positive integer x so that tan tan 1


tan 1
= tan , is
4
10
x 1

(A) 8
(B) 9
(C) 7
(D) 0

4.

When (x1/4 x2/3)7 is multiplied out and simplified, one of its terms has the form kx3 where 'k' is a constant
which is equal to
(A) 7
(B) 7
(C) 35
(D) 35

5.

If x =

6.

Number of solutions of the trigonometric equation cos 3x 3cos x sin2x = cos3x where x (0, 1) is
(A) 0
(B) 1
(C) 2
(D) infinite

7.

In which one of the following cases, limit does not tend to e ?

1 i 3
1 i 3
and y =
, where i2 = 1, then which of the following is not correct ?
2
2
(A) x5 + y5 = 1
(B) x7 + y7 = 1
(C) x9 + y9 = 1
(D) x11 + y11 = 1

(B) Lim 1
x
x

(A) Lim x x 1
x 1

x 4

(C) Lim
x x 2

x 3

(D) Lim ( f f ( x )) f ( x ) when Lim f(x) 0


x

8.

The line L and K are symmetric to each other with respect to the line y = x. If the equation of the line L is
y = ax + b where a and b are non zero, then the equation of K is
(A) y =

x
b

a
a

(B) y =

x
b
a

(C) y =
3x 4x

x
b
+
a
a

x
b
+
a
a

9.

Domain of definition of the function f(x) =

10.

The roots of x2 + bx + c = 0 are both real and greater than 1. If s = b + c + 1, then 's'
(A) may be less than zero
(B) may be equal to zero
(C) must be greater than zero
(D) must be less than zero

11.

Which one of the following does not reduce to sinx for every x where the expressions are defined ?

(A) ( , 0]

(A)

sin x

(B) [0, )

sec x tan x
2

x 2 3x 4

(B) cosx cot x cos x

is

(D) y =

(C) ( , 1) [0, 4)

(C)

sin 2 x sec x
tan x

(D) ( , 1) (1, 4)

(D) all reduce to sin x

Extra Edge Assignment

Subject : Mathematics
1.

Assignment 12

Let f(x) be a function with two properties


(a)
for any two real number x and y, f(x + y) = x + f(y) and
(b)
f(0) = 2
The value of f(100) is
(A) 2
(B) 98
(C) 102

(D) 100

2.

Read the following statements carefully :


.
If a, b and c are positive numbers not equal to 1 and a < b, then log ac < logbc.
.
The equation x2 b = 0 has a real solution for x for any real number b.
.
the sequence an defined by an = 3(0.2)n is a geometric sequence.
V.
cos(cos(x)) < 1/2, x R
Now indicate the correct alternative
(A) exactly one is always true
(B) exactly two are always true
(C) exactly three are always true
(D) exactly four are always true

3.

If x = a + bi is a complex number such that x2 = 3 + 4i and x3 = 2 + 11i where i =

4.

If x satisfies log2x + logx2 = 4, then log2x can be equal to

(A) 2

(A) tan
5.

7.

12

abc

4R

9.
10.

12

(B)

2
R

(C)

R(a b c )
r

Let z = (0, 1) C. Where C is the set of complex numbers, then the sum
(B) i

(D) cot

(D)

2rs
R

(C) 0

z
n

k 0

for n N can be equal to

(D) 1

Value of the expression log1/2(sin 6. sin42.sin45.sin66.sin78)


(A) lies between 4 and 5
(B) is rational which is not integral
(C) is irrational which is a simple surd
(D) is irrational which is a mixed surd.

1 ax

has the value equal to unity then 'a' is equal to


If a > 0 and xLim
0 1 2 x
(A) 1
(B) 2
(C) 3
(D) 4
1/ x

The first three terms of a geometric sequence are x, y, z and these have the sum equal to 42. If the middle
term y is multiplied by 5/4, the number x,
value of x, is
(A) 6

11.

(C) tan

(D) 5

In a triangle ABC if B = 30, b = 3 2 6 and c = 6 2 3 then


(A) the triangle ABC is an obtuse triangle
(B) angle A can be 15
(C) there can be only one value for the side BC (D) the value of tan A tan C will be unique.

(A) 1 + i
8.

(B) cot

(C) 4

In a triangle ABC, altitude from its vertex meet the oppositve sides in D, E and F. Then the perimeter of the
triangle DEF, is
(A)

6.

(B) 3

1 , then (a + b) equal to

(B) 12

5y
, z now form an arithmetric sequence. The largest possible
4
(C) 24

The value of the expression sin2 1 + sin2 2 + sin2 3 + ...... + sin2 90


(A) 0
(B) 45
(C) 45.5

(D) 30
(D) 90

Extra Edge Assignment

Subject : Mathematics

Assignment 13

1.

In a triangle ABC with altitude AD, BAC = 45, DB = 3 and CD = 2. The area of the triangle ABC is
(A) 6
(B) 15
(C) 15/4
(D) 12

2.

When the polynomial 5x3 + Mx + N is divided by x2 + x + 1 the remainder is 0. the value of (M + N) is equal
to
(A) 3
(B) 5
(C) 5
(D) 15

3.

Number of real values of x for which the area of the triangle formed by 3 points A(2, 1) ; B(1, 3) and
C(3x , 2x 3) is 8 sq. units is
(A) 0
(B) 1
(C) 2
(D) infinitely many

4.

Assume that p is a real number. In order for

5.

Find the equation of the circle which has its diameter the chord cut off on the line px + qy 1 = 0 by the circle
x2 + y2 = a2

6.

Obtain a relation in a and b, if possible, so that the function

(A) p 1/4

f(x) = Lim
n
7.

(B) p 1/4

x n (a sin( x n )) (b sin( x n ))
(1 x n ) sec(tan 1( x n x n ))

x 3p 1

x = 1 to have real solutions, it is necessary that


(C) p 1/3
(D) p 1/3
3

is continuous at x = 1

The interior angle bisector of angle A for the triangle ABC whose coordinates of the vertices are A( 8, 5) ;
B(15, 19) and C(1, 7) has the equation ax + 2y + c = 0. Find 'a' and 'c'
In ABC (a + b) (a b) = c(b + c), the measure of angle A, is
(A) 30
(B) 60
(C) 90

(D) 120

9.

The point A (sin , cos ) is 3 units away from the point B(2 cos 75, 2 sin 75). If 0 < 360. Then is
(A) 15
(B) 165
(C) 195
(D) 255

10.

The radius of the circle inscribed in a triangle with sides 12, 35 and 37 is
(A) 4
(B) 5
(C) 6

(D) 7

11.

Consider the equation 10z2 3iz k = 0, where z is a complex variable and i2 = 1. Which of the following
statements is True ?
(A) For all real positive numbers k, both roots are pure imaginary
(B) For real negative real numbers k, both roots are pure imaginary
(C) For all pure imaginary numbers k, both roots are real and irrational
(D) For all complex numbes k, neither roots is real

12.

The set of values of x for which the function defined as

8.

1 x

(1 x )
f(x) =
3 x

x 1

(2 x )

1 x 2
x2

fails to be continuous or differentiable, is


(A) {1}
(B) {2}
13.

(C) {1, 2}

The diagram shows several numbers in the complex plane. The


circle is the unit circle centered at the origin. One of these
numbers is the reciprocal of F, which is
(A) A
(B) B
(C) C
(D) D

(D)

Imaginary
axis

F
real axis

Extra Edge Assignment

Subject : Mathematics

1.

Assignment 14

A triangle has side a = 7 , the oppositve angle a = 60, and the sum of the two other sides is (b + c) = 5.
The ratio of the longest to the shortest side of the triangle is
(A) 1

(B)

(C)

3
2

(D)

7
2

2.

Evaluate : Lim n2 (n a n 1 a ) (a > 0, n N)

3.

Show that the centroid of the triangle of which the three altitudes to its sides lie on the line y = m 1x ; y = m2x
and y = m3x lie on the line, y (m 1m2 + m2m3 + m3m1 + 3) = (m1 + m2 + m3 + 3m1 m2 m3) x.

4.
5.

Use of series expansion and L'Hospital's rule is not allowed

Find the equations of the circles which touch the co-ordinate axes and the line 3x + 4y = 12
f(x) =

2 cos x sin 2 x
( 2x )

; g(x) =

e cos x 1
8 x 4

= f(x) for x < /2


= g(x) for x > /2
then which of the following holds ?
(A) h is continuous x = /2
h(x)

(C) h has a removable discontinuity at x = /2


6.

7.

(B) h has an irremovable discontinuity at x = /2


(D) f
2

=g

Two ballss are drawn from a bag containing 3 white, 4 black and 5 red balls then the number of ways in which
the two balls of different colours are drawn is
(A) 94
(B) 47
(C) 38
(D) 19
If ABC if cosA, cosB, cosC are in A.P. then which of the following is also an A.P. ?

A
B
C
, tan , tan
2
2
2
(C) (s a) (s b), (s c)
(A) tan

A
B
C
, cot
, cot
2
2
2
(D) none
(B) cot

8.

If tan1(x) + tan1(2x) + tan1(3x) = , then


(A) x = 1
(B) x = 0

9.

The most general solutions of the equation x3 sin 2x + 2 =


(A) x = n (1)n
where n

12

(B) x =

(1)n
2
12

(C) x = 1

(D) x

x is

(C) x = 0

(D) x = n (1)n

12

10.

The sum of thesquare of the length of the chords intercepted by the line x + y = n, n N on the circle
x2 + y2 = 4 is
(A) 11
(B) 22
(C) 33
(D) none

11.

Which one of the following statements about the function y = f(x) ,


graphed have is true ?
(A) Lim
f(x) = 0
x 1

(C) xLim
x 0 f(x) exists at every point x0 is (1, 1)

(B) Lim
f(x) = 1
x 1

(D) Lim
f(x) = 1
x 1

y = f(x)
1

Extra Edge Assignment

Subject : Mathematics
1.

x
2
Lim a sin bx b sin ax (a b) is
x 0
tan bx tan ax

(A) 1
2.

Lim
x 0

(A)

2
3

(B)

ab
ab

(C)

ab
ab

(D) non existant

(B)

4
3

(C)

3
4

(D)

2x 2
is
3 3 cos x

x2 9

Assignment 15

x2 9

3.

Let = Lim
x 3

4.

If (2 x2) g(x) 2 cos x for all x, then Lim


g(x) is equal to
x 0

5.

Let f(x) =

6.

7.

8.

9.

x2 7 4

(A) m
(A) 1

11.

(B) = 2m
(B) 2

(A) 5

then
x2 7 4
(C) = m
(C) 1/2

3x 2 (7x 1) x 5
, then Lim
f(x) is
x 0
x 1
(B) 2
(C) 6

(D) = m
(D) 0

(D) non existent

Lim (nx ) x e is

x e

(A) e 1/e

Lim (ex + x)1/x


x 0

(A) e1

Lim

n( x 1)
log 2 x is

(A) log2e

Let = Lim
/ 3
(A) equal to 3

10.

and m = Lim
x 3

1
3

(B) ee

(C) e2

(D) e1

(B) e1/2

(C) e3/2

(D) e2

(B) 0

(C) n2

(D) non existent

sin , then [] is, where [ ] indicates greatest integer function


3

x3
sin x e 3
Lim e
x 0
x
(A) e + 1

(B) equal to 2

(C) equal to 1

(D) none existent

(B) e 3 + 1

(C) e3 + 1

(D) e3 1

Lim cos(tan1(sin (tan1x))) is equal to

(A) 1

(B)

(C)

(D)

Extra Edge Assignment

Subject : Mathematics
1.

2.

Let f(x) =

ax 2 b

(A) 1

x2 1

Assignment 16

, if Lim
f(x) = 1 and Lim
f(x) = 1, then f(2) + f(2) is equal to
x 0
x
(B) 2

(C) 0

(D) 4

x n 3n
If Lim
= 108 (n N) then the value of n is
x 3
x3
(A) 3
(B) 4
(C) 5

(D) 6

tan 2x 2 sin x

3.

Find the limiting value of

4.

Without usingseries expansion or L'Hospital's rule evaluate Lim


x 0

5.

Show that the sum of infinite series ,


tan1

6.

as x tends to zero

x3

n (1 x 2 x 4 )
(e x 1) x

4
4
4
4

+ tan1
+ tan1
+ tan1
+ ....... =
+ cot1 3
19
39
67
7
4

3x
Draw the graph of the function f(x) =
4 x
at x = 1

,
,

1 x 1
1 x 4

and discuss its continuity and defferentiability

x
. Find the largest value of 'a', for which f has an inverse function
2
f1. Find f1. State the domain and the range of f and f1. Find the gradient of the curve y = f1(x) at the point
where the crosses the y-axis.

7.

Given f : [0, a] S, such that f(x) = 3 cos

8.

Given f(x) = [x] tan (x) where [x] denotes greatest integer function, find the LHD and RHD at x = k,
where k

9.

Examine the continuity at x = 0 of the sum function of the infinite series

x
x
x
+
+
+ .........
(
x

1
)(
2
x

1
)
(
2
x

1
)(
3x 1)
x 1
1 nt

10.

If x =

11.

Let f(x) = tan

t2

3 2nt
dy
dy
and y =
. Show that y
= 2x + 1
t
dx
dx
2

x
x
x
x
x
x
x
sec x + tan 2 sec
+ tan 3 sec 2 + ......+ tan n sec n 1
2
2
2
2
2
2
2

and g(x) = f(x) + tan

g( x )

(a) Lim
x 0
x
12.

1/ x


where x , and n N. Evaluate the following limits.
2 2

g(x)
(b) Lim
x 0 x

1/ x 2

g(x)
(c) Lim
x 0 x

Without using L'Hospitals rule or series expansion evaluate : Lim


x 0

1/ x 3

x n x 2 1 x

3
x

Extra Edge Assignment

Subject : Mathematics
1.

Suppose f(x) = tan (sin1(2x))


(a)
Find the domain and range of f
(b)
Express f(x) as an algebaric function of x
(c)

2.

3.
4.

Assignment 17

1
Find f .
4

x(3e1/ x 4)

1/ x
Discuss the limit, continuity and differentiability of the function f(x) = 2 e
0

,
,

x0
x0

at x = 0.

ntan ax
4
(b 0). Use of series expansion and L'Hospital's rule is not allowed

Evaluate : Lim
x 0
sin bx
The function f is defined by y = f(x). Where x = 2t |t| , y = t2 | t |, t R. Draw the graph of f for the interval
1 x 1. Also discuss its continuity and differentiability at x = 0

5.

1
1

. Use of series expansion or L'Hospital's rule is not allowed


Evaluate Lim
x 0 nx
x 1

6.

If g is an inverse function of f and f(x) =

7.

Given a real valued functions f(x) as follows

x 2 2 cos x 2

x4

p
f(x) =
sin x n(e x cos x )

6x 2

1 xn

for

x0

for

x0

for

, prove that g(x) = 1 + [g(x)]n

x0

Determine the value of p if possible, so that the function is continuous at x = 0. Use of power series or
L'Hospital's rule is not allowed.
dy
sin 2 x
cos 2 x

+
, then
at x =
is
dx
1 cot x
1 tan x
4
(A) 0
(B) 1
(C) 1

8.

If y =

9.

1
1 1
The value of cot 2 3 tan 3 equals


(A) (10 3 )1

10.

(B) (3 10 )1

(C) (3 10 )

sin 1 cos 1 x

Lim
equals
2
x 1
1 x
(A) 0

(B) 1

(D) 2

(C)

(D) (10 3 )

(D)

Subject : Mathematics
1.

1 7 1 x
If f(x) = 3 +

Extra Edge Assignment

Assignment 18
then

(A) xLim
f(x) = 4
1

(B) xLim
f(x) = 3
1

(C) xLim
f(x) = 5
1
2.

f (1 h) f (1)

If f(x) = 3x10 7x8 + 5x6 21x3 + 3x2 7, then the value of xLim
1
(A)

3.

(D) f has irremovable discontinuity at x = 1

53
3

(B)

22
3

(C)

h3 3h

53
3

is

(D)

22
3

If the triangle formed by the lines x2 y2 = 0 and the line x + 2y = 1 is isosceles then =
(A) 1
(B) 2
(C) 3
(D) 0

4.

(e 2 x 1 2x 2 )(cos x 1)
Using L'Hospital's rule or otherwise evaluate the limit, Lim
x 0
(sin 3 x n(1 3 x )x 4

5.

e x n ( x e )
Evaluate the limit Lim
. Use of L'Hospital's rule or surd expansion not allowed.
x 0
ex 1

6.

Find all real numbers t satisfying the equation (3t 9)3 + (9t 3)3 = (9t + 3t 12)3

7.

Find g(3) if g(x) = x . 2h(x) where h(3) = 2 and h(3) = 5.

8.

Find the value of the expression

9.

Let f(x) = a cos(x + 1) + b cos(x +2) + c cos(x + 3), where a, b, c are real. Given that f(x) has atleast two
zeros in the interval (0, ), find all its real zeroes.

10.
11.

log 4 ( 2000 )

log 5 ( 2000 ) 6

1
63

Calculate, sin 4 arcsin 8 .

In an infinite pattern, a square is placed. i nsdie a square, as


shown, such that each square is at a constant angle to its
predecessor. the largest outermost square is of side unity. Find
the sum of the areas of all the square in the infinite pattern as a
function of .

12.

If is eliminated from the equations, a cos + b sin = c and a cos2 + bsin2 = c, show that the eliminant
is (a b)2 (a c) (b c) + 4a2b2 = 0.

13.

A triangle has side lengths 18, 24 and 30. Find the area of the triangle whose vertices are the incentre
circumcentre and centroid of the triangle.

Extra Edge Assignment

Subject : Mathematics

1.

Assignment 19

Find the real solutions to the system of equations


log10(2000 xy) log10x . log10y = 4
log10(2yz) log10y . log10z = 1
and
log10(zx) log10z . log10x = 0

2.

1 cos x
x

1
Prove that, cos1 1
= 2 cot1 tan where x (0, )

2
12 cos x 13
5

3.

1
1 24

Compute the value of cos tan


7
4

4.

If g(x) = x3 + px2 + qx + r where p, q and r are integers. If g(0) and g(1) and both odd, then prove that the
equation g(x) = 0 cannot have three integer roots.

5.

Sum the series


cot1(2a1 + a) + cot1(2a1 + 3a) + cot1(2a1 + 6a) + cot1(2a1 + 10a) + ...... + to 'n' terms.
Also find the sum of infinte terms, (a > 0).

cos n
44

6.

Let x =

n 1
44

n 1

7.

sin n o

find the greatest integer that does not exceed 100 x.

The set of all x for which

10
3
> 2
consists of the union of a finite and an infinite interval. The length of the
x
x 1

(A) 3

(B) 2

finite interval is

1
3

(C)

1
3

(D) 2

2
3

8.

Five persons put their hats in a pile. When they pick up hats later. each one gets some one else's hat.
Number of ways this can happen is
(A) 40
(B) 44
(C) 96
(D) 120

9.

Suppose the origin and the point (0, 5) are on a circle whose diameter is along the y-axis and (a, b) lies one
the circle. Let L be the line that passes through the origin and (a, b). If a2 + b2 = 16 and a > 0 then the equation
of the line L is
(A) 3x 4y = 0
(B) 20x 3y = 0
(C) 2x y = 0
(D) 4x 3y = 0

10.

4[ x]

If lies between the roots of the equation y2 my + 1 = 0 then the value of | x |2 16 x R


has the value equal to (Here [x] denotes greatest integer function)
(A) 0
(B) 1
(C) 2

(D) none

11.

The sum of the squares of the three solutions to the equation x3 + x2 + x + 1 = 0, is


(A) 1
(B) 0
(C) 1
(D) 2

12.

Let f(x) = 1 + x3. If g(x) = f1(x), i.e. if g is the inverse f, then g(9) equal to
(A) 1/12
(B) 1/243
(C) 1/8
(D) 1/24

Extra Edge Assignment

Subject : Mathematics
1.
2.

3.

Assignment 20

Lim x x x x is
x

(A) equal to 0
(B) equal to 1

x 1

If g x 1 = 3x then the vlaue of g(3), is

15
4

2
2

(B)

4.

For acute angles A and B if (tan A) (cot B) =

5.

The value of this product of 98 numbers

(A) 7/4

1
10

(D)

(B)

98
100

(A)

(B)

2 4e T / 3

Evaluate :

x 8

12 x 35

2 4e T / 3

(B) 1.5

(C)

1
5050

(C)

(C)

x3
3x 2
+
x+C
3
2

(B)

x3
3x 2

x+C
3
2

2 4e T / 3

(C) 1.75

Let f be a polynomial function such that for all real x


f(x2 +1) + x4 + 5x2 + 3
then the value of f(x) w.r.t.x, is
(A)

(D)

10.

Suppose f is a differentiable functions such that for every real number x,


f(x) + 2f(x) = sinx, then f(/4) has the value equal to

11.

(B)

(D)

1
4950
2 4e T / 3

(D) 2

x3
3x 2
+
+x+C
3
2

Number of regular polygons that have integral interior angle measure, is


(A) 20
(B) 21
(C) 22

(D)

x3
3x 2

+x+C
3
2

9.

(A)

3
3

sin( A B)
9
then the value of
equal to
sin(A B)
5
(C) 4/7
(D) 7/2

If T = 3 n (x2 + x) with > 0 and x > 0, then 2x + is equal to

(A) 1.25
8.

(B) 2/7

(C) 9

2
2
2
2
2
2

1 1 1 ......... 1
1
1
, is
4 5
3
98 99 100

(A)

7.

(D) equal to 1/2

Suppose f is a differentiable function such that f(x + y) = f(x) = f(y) + 5xy for all x, y and f(0) = 3. The minimum
value of f(x) is
(A) 1
(B) 9/10
(C) 9/25
(D) none

(A)

6.

(C) equal to 1

(C)

(D) 23

(D) 2
2
2
2 2
the number of permutatio9n of the letters AAAA BBBC in which the A's appear together in a block of four
letters or the B's appear in a block of 3 letters, is
(A) 44
(B) 50
(C) 60
(D) none

Extra Edge Assignment

Subject : Mathematics

Assignment 21

1.

If {x} denotes the fractional part function then the number x =

2.

Which of the following is wrong ?

(A) 1/2

(A)
(C)

3.

(B) 0

tan sec 2 d

tan 2
C
2

tan kx

x
Let f(x) =
2
3 x 2k

x0

for

x0

for

{ 3 } 2 2{ 2 } 2

simplifies to

(D) none

tan sec 2 d

sec 2
C
2

(D) none

. If f(x) is continuous at x = 0, then the number of values of k is

(B) 1

1
1
1

Find Lim
y 2 y 2 x y 2
x

(A) 0

5.

(B)

x sin xdx sin x x cos x C

(A) none
4.

(C) 1/2

{ 3 } 2{ 2 }

(B) n x

(C) 2

(D) more than 2

(C) 1/x 2

(D) does not exist

Let p(x) be the cubic polynomial 7x3 4x2 + K. Suppose the three roots of p(x) form an arithmetic progression. Then the value of K, is
(A)

4
21

(B)

16
147

(C)

16
441

(D)

128
1323

6.

The sum (in radians) of all values of x with 0 x 2 which satisfy 2 (cos 2x sin x 1) = 1 + 2 sinx, is
(A) 2
(B) 3
(C) 4
(D) 6

7.

The value of Lim


n
(A) 1

1 2
n

n 0

2n is

(B) 2

(C) 4

(D) none

dy
at the point (0, ) must be
dx
(C) 0
(D) 2

8.

If sin (x + 2y) = 2x cos y, then the value of

9.

For x > 0 and 1 and n N, evaluate,

10.

Show that (a + b + c), (a2 + b2 + c2) are the factors of the determinant

(A) 1

1
1
1
Lim

.......
n

n log x 2. log x 4
logx 4. logx 8
logx 2 1. logx 2n

a2

b
c

11.

(B) 3/2

2
2

(b c )2

bc

(a b )

ab

(c a ) 2
2

ca

. Also find the remaining factors.

Prove that a non singular idepmotent matrix is always an involutary matrix.

Extra Edge Assignment

Subject : Mathematics

Assignment 22

57

27

1.

8
Find an upper triangular matrix. A such that A3 =
0

2.

If 'y' is a twice differentiable function of x, transofrm the equation (1 x 2)


the transformation, x = sin t, in terms of the independent variable 't'.

3.

d2 y

dx

dy
+ y = 0 by means of
dx

A tangent line is drawn to a circle of radius unity at the point A, and a segment AB is laid off whose length is
equal to that of the arc AC. A straight line BC is drawn to intersect the extension of the diameter AO at the
point P. Prove that :
(1 cos )
Lim PA = 3
(ii)
0
sin
Use of series expansion of L'Hospital's rule prohibited.

(i)

4.

PA =

1x

e1 1
Lim
Without using any series expansion or L'Hospital's rule, Evaluate : x x n
x

13 3

15 26

2 5
5

10 .
5

5.

Find the value of the determinant

6.

f(x) is a differentiable function satisfy the relationship f2(x) + f2(y) + 2(xy 1) = f2(x + y) x, y R

7.

x
x
x

+ tan1
+ tan1
+ ...... upto n terms.
Let, y = tan1
2
2
2
1
.
2

x
2
.
3

x
2
.
3

3 65

15

Also f(x) > 0 x R, and f ( 2 ) = 2. Determine f(x).

Find

dy
expressing your answer in two terms.
dx

xa

x b

xb

xc

xa

xc

8.

Without expanding the determinant show that the equation

9.

Let1, 2 and 1, 2 be the roots of ax2 + bx + c and px2 + qx + r = 0, respectively. If the system of equations
1y + 2z = 0 and 1y + 2z = 0 has a non-trivial solution, then prove that

b2
q

ac
pr

= 0 has zero as a root.

Extra Edge Assignment

Subject : Mathematics

Assignment 23

1.

x 0 2x1
Compute xn in terms x0, x1 and n. Also evaluate Lim
x =
n n
3

2.

2
A=
b

a
5
8

c
is Symmetric and B =
2

b a
2

3
e
6

2b c
is Skew symmetric, then find AB.
f
a

Is AB a symmetric, Skew Symmetric or neither of them. Justify your answer.


3.

4.

x 2 1
e x

Let f(x) =
,x<0
=0
x=0
= x2 ,
x>0
Discuss continuity and differentiability of f(x) at x = 0
1
Show that the matrix A =
2

1
evaluate the matrix
2

5.
6.

7.

Find

2007

0
can be decomposed as a sum of a unit and a nilpotent matrix. Hence
1

dy
, if (tan1x)y + ycot x = 1
dx

f (( h)3 ) f ( 3 )
If f is differentiable and Lim
= , then find the value of f(3), Without using L'Hospital's
h0
h
rule.

x
1 e

Consider the function f(x) = x 2


3

6 x

,
,
,

x0

0x3
x3

(A) Find all points where f(x) is discontinuous.


(B) Find all points when f(x) is not differentiable
(C) Draw the graph , showing clearly the points of discontinuity or non derivability.
8.

9.

A circle of radius 2 has center at (2, 0). A circle or radius 1 has center at (5, 0). A line is tangent to the two
circles at points in the first quadrant. Which of the following is the y-intercept of the line ?
8
2
(C)
(D) 2 2
3
4
In a triangle ABC, the length of AB is , the length of BC is 5, and the length of CA is 4. if K lies on BC such

(A) 3

that the ratio of length


(A) 2 3
10.

(B)

BK
3
is , then the lenght of AK is
KC
2

(B) 4

(C) 3 2

(D) 2

21
5

Which one of the following quadrants has the most solutions to the inequality, x y < 2 ?
(A) quadrant
(B) quadrant
(C) quadrant
(D) and quadrant have same

Extra Edge Assignment

Subject : Mathematics

1.

3
(B) ,

4 4

(A) (0, )
2.
3.

5.

6.

(C) [0, ]

(D) R

The area of the region of the plane consisting of all points whose coordinates (x, y) satisfy the conditions
4 x2 + y2 36 and y |x| , is
(A) 24
(B) 27
(C) 20
(D) 32

A straight wire 60 cm long is bent into the shape of an L. the shortest possible distance between the two
ends of the bent wire, is
(A) 30 cm

4.

Assignment 24

The range of the function f(x) = sin x + tan1x + cos1x, is


1

(B) 30 2 cm

(C) 10 26

(D) 20 5


Sum of values of x, in 0 , for which tan x = 9 tan x holds, is
4

2
1
1
(A) 0
(B) tan (2)
(C) cot (0)
(D) tan1(2)
3
= e and f(0) = 1, then f(x) equals
Given f (x) = cos x, f
2
(A) sinx (e + 1) x
(B) sinx + (e + 1) x
(C) (e + 1) x + cos x

Evaluate the integral :


(A)

1 cos x 2
e
+C
2

xe

cos x 2

(B)

x 2 kx 1

(D) (e + 1) x cos x + 2

sin x 2 dx

1 sin x 2
e
+C
2

(C)

1 sin x 2
e
+C
2

(D)

1 cos x 2
e
+C
2

. The intervals of all possible values of k for which f is continuous for every x R, is

7.

Let f(x) =

8.

From a point P outside of a circle with centre at 0, tangent segments PA and PB are drawn.

(A) ( , 2]

If

( AO )

(A) 6
9.

Let

1 =

x2 k

(PA )

a11

a 21
a31

b11

(C) R ( 2, 2)

1
, then the length of the chord AB is
16

a12

a 22
a32

b12

(B) 4

a13

a 23
a33

(C) 8

, 1 0

b13

b 22

b 23 where cij is cofactor of aij i, j = 1, 2, 3

c11

c 12

c 13

3 =

c 21
c 31

b 32
c 22
c 32

b33

c 23
c 33

where cij is cofactor of bij i, j = 1, 2, 3

then which one of the following is always correct.


(A) 1, 2 , 3 are in A.P.
(B) 1, 2, 3 are in G.P.

3
(C) 12 =
2

(D) 1 =

(D) (2, 2)

(D) 9

2 = b 21

b31

and

(B) [2, 0)

Extra Edge Assignment

Subject : Mathematics

1.

2.

The first three terms of an arithmetic sequence, in order, are 2x + 4, 5x 4 and 3x + 4. The sum of the first
10 terms of this sequence, is
(A) 176
(B) 202.4
(C) 352
(D) 396

i sin is equal to
The value of cos i sin cos
8
8
15
15

(A)

3.

4.
5.

Assignment 25

i
3
+
2
2

Evaluate :

(B)

i
3
+
2
2

(C)

i
3

2
2

(D)

i
3

2
2

dx

ax x 2

e e x
The value of Lim
is
x
sin x
(A) 0
(B) e

(C) e

(D) non exitent

Suppose F(x) = f(g(x)) and g(3) = 5, g(3) = 3, f(3) = 1, f(5) = 4. Then the value of F(3), is
(A) 15
(B) 12
(C) 9
(D) 7

6.

sin

7.

tan =

8.

Find the sum of the solutions of the equation 2e2x 5ex + 4 = 0.

9.

Suppose that x and y are positive numbers for which log9x = log12y = log16(x + y). If the value of

sin1 x cos 1 x
x cos 1 x

1
2

1
2

where (0, 2), find the possible value of


where 0 , find .
2

10.

Consider f(x) =

ba

ba
sin 2 x
a

ba

1
sin x

y
= 2 cos,
x

a b tan 2 x , for b > a > 0 and the functions g(x) and h(x) are

f(x)
and h(x) = sgn (f(x)) for x domain of 'f', otherwise g(x) = 0 = h(x) for
defined such that g(x) = [f(x)]
2
x domain of 'f', where [x] is the greatest integer function of x and {x} is the fractional part of x. Then discuss

the continuity of 'g' and 'h' at x =


11.

(1 x

x 2 tan 1 x
)

2 5/2

dx

and x = 0 respectively..
2

Extra Edge Assignment

Subject : Mathematics

Assignment 26

cos ec

x dx

1.

Using substitution only, evaluate :

2.

If sin A =

3.

The straight line

4.

If

5.

If a ABC is formed by the lines 2x + y 3 = 0 ; x y + 5 = 0 and 3x y + 1 = 0, then obtain a cubic equation


whose roots are the tangent of the interior angles of the triangle.

6.

Integrate :

7.

12
A
. Find the value of tan
13
2

x y
= 1 cuts the x-axis and y-axis in A and B respectively and a straight line perpendicua b
lar to AB cuts them in P and Q respectively. Find the locus of the point of intersection of AQ and BP.
tan
1
cot
= , find the value of
tan tan 3
3
cot cot 3

(a

(a

dx

tan x ) b 2 tan 2 x
2

x sin x cos x

cos 2 x b 2 sin 2 x )2

(a > b)

dx

d 2
dy
(x y) = x 1 where x 0 and y = 0 when x = 1. Find the set of values of x for which
is positive
dx
dx

8.

Let

9.

Let x = (0.15)20. Find the characteristic and mantissa in the logarithm of x, to the base 10. Assume log 102 =
0.301 and log103 = 0.477

10.

11.

Two circles of radii R and r are externally tangent. Find the radius of the third circle which is between them
and touches those circles and their external common tangent in terms of R and r.

x 5 x 5 5x
Let a matrix A be denoted as A = diag. 5 , 5 , 5 then compute the value of integral

(det A )dx .

12.

Using albebraic geometry prove that in an isosceles triangle the sum of the distances from any point of the
base to the lateral sides is constant
(You may assume origin to be the middle point of the base of the isosceles triangle)

14.

Evaluate

15.

Integrate :

1 x
1 x

dx

x x2 x3

tan x dx

Extra Edge Assignment

Subject : Mathematics
1.

nx n x 2 1 x
The value of Lim
x

(A)

2.

Assignment 27

1
2

Evaluate :
(A) 1/4

(tan x. sec

/4
0

1
(B) n
2

(C) does not exist

(D) 0

(C) 3/4

(D) 1

x ) dx

(B) 1/2

3.

The product of two positive numbers is 1. The smallest possible value of sum of their squares is

4.

Given that log(2) = 0.3010 ........, number of digits in the number 2000 2000 is
(A) 6601
(B) 6602
(C) 6603
(D) 6604

5.

Given that a, b and c are the roots of the equation x3 2x2 11x + 12 = 0, then the value of

(A) 25

(A)
6.

If

(B) 24

5
6

(B)

(C) 18 2

11
12

(C)

tan x dx 2 , then b is equal to


b
0

(A) arc cos(2e)

(B) arc sec(2)

13
12

(C) arc sec2(e)


= 8( x 3 x 2) , is
(C) 3

(D) 18

(D)

7
6

1
1
1
+
+
a
b
c

(D) none

7.

The sum of all values of x so that 16( x


(A) 0
(B) 3

8.

A certain function f(x) satisfies f(x) + 2 f(6 x) = x for all real numbers x. The value of f(1) is
(A) 3
(B) 2
(C) 1
(D) not possible to determine

9.

Number of ways in which the letters A, B, C and D be arranged in a sequence so that A is not in position 3,
B is not in position 1, C is not in position 2 and D is not in position 4, is
(A) 8
(B) 15
(C) 9
(D) 6

10.

Using only the letter from the word WILDCATS with no repetitions allowed in a codeword, number of 4 letter
codewords are possible that both start and end with a consonant, are
(A) 360
(B) 900
(C) 1680
(D) 2204

11.

12.

13.

Find

( xnx ) dx

(A)

1
4

3 x 1)

(B)

1
2

(C) 1

If P(x) is a polynomial with rational coefficients and roots at 0, 1,


at least
(A) 4

(B) 5

Sum of the infinite series,


(A)

1
7

(D) 5

(C) 6

(D) 1
2 and 1

3 , then the degree of P(x) is

(D) 8

5
1
5
1
5
1
2 + 3 4 + 5 6 + ......... , is equal to
7
7
7
7
7
7

(B)

1
24

(C)

1
49

(D)

1
5

Extra Edge Assignment

Subject : Mathematics
Assignment 28
1.
A florist has in stock several dozens of each of the following : roses, carnations, and lilies. How many
different bouquets of half dozen flowers can be made ?

8!
(A) 2!. 6!

2.

9!
(B) 3!. 6!

e3x 1
Let f(x) = x

3
(A) 9

if
if

x0

x0
(B) 9

12!
(C) 6 !. 6!

(D) 56

(C) 9/2

(D) non existent

, then f (0) , is

3.

If f (x) = 10 and f (1) = 6 and f(1) = 4, then ( 1) is equals


(A) 4
(B) 2
(C) 8

4.

In how many ways can six boys and five girls stand in a row if all the girls are to stand together but the boys
cannot all stand together ?
(A) 172,800
(B) 432,000
(C) 86,400
(D) none

5.

The composite of two functions f and g is denoted by fog and defined by (fog)(x) = f(g(x)). When f(x) =
and g(x) =
(A)

6.

4x
x2

(D) 12

5x
which one of the following is equal to (fog) (x) ?
x2
(B)

30 x
5x 2

(C)

30 x
5x 2

(D)

6x
x 1

15 x
2x 1

1 1

k 1/ k
The equation n
= F(k). n1 k 1 k nk is true for all k whrever defined. F(100) has the

(k 1)1/(k 1)

value equal to
(A) 100

(B)

1
101

1)3

2x

(C) 5050

(D)

1
100

dx

7.

Compute :

8.

Let y =

9.

Show that cos2A + cos2(A + B) + 2 cosA cos(180 + B) .cos(360 + A + B) is independent of A. Hence find its
value when B = 810.

log

x 3 x

21 / 4

3 log27 ( x

7 4 log49 x x 1

and

dy
= ax + b, find the value of a and b.
dx

10.

Find the product of the roots of the equation , |x|2 + |x| 6 0

11.

One root of mx2 10x +3 = 0 is two third of the other root. Find the sum of the roots.

12.

Suppose x and y are real numbers such that tan x + tan y = 42 and cot x + coty = 49. Find the value of tan
(x + y).

13.

Find the solution set of k so that y = kx is secant to the curve y = x2 + k.

Extra Edge Assignment

Subject : Mathematics

1.

A quadratic polynomial p9x) has 1 +

2.

Solve the equation x

polynomial.

0.5 log

( x2 x)

Assignment 29

5 and 1

5 as roots and it satisfies p(1) = 2. Find the quadratic

= 3 log9 4 .

3
5
+ cos
+ cos
+ ....... upto n terms. Do not use any
2n 1
2n 1
2n 1

3.

Find the sum of the series , cos

4.

Find the minimum and maximum value of f(x, y) = 4x2 + 4xy + 3y2 subjcected to x2 + y2 = 1.

direct formula of summation.

5.

Find the minimum and maximum value of (sinx cosx 1) (sinx + cosx 1) x R.

6.

Given that log2a = s, log4b = s2 and logc 2 (8) = logc 2 (8) . Write log2
(a, b, c > 0, c 1).

x 2 2x 8

a 2b 5
c4

as a function of 's'

7.

Find the range of the expression y =

8.

Find whether a triangle ABC can exists with the tangents of its interior angle satisfying, tanA = x,
tan B = x + 1 and tan C = 1 + x for some real value of x. Justify your assertion with adequate reasoning.

9.

Solve the equation, 5 sin x +

10.

Find the value of x, y, z satisfying the equations


log2x + log4y + log4z = 2
log9x + log3y + log9z = 2
and
log16x + log16y + log4z = 2

11.

Integrate :

12.

Find the domain of definition of the function, f(x) = log4 log3(log2(x2 2x + 3) log

13.

Integrate :

x9
3

9x

2 x

1 x 2
2

dx

1
5
5 = 2 sin2x +
if x (0, ).
2 sin x
2 sin2 x

(2x 1))

dx .

14.

Evaluate :

15.

Examine the function f(x) = Lim


n

, for all permissible value of x.

dx

2 cos 2x

x 2 4x 5

1 ( 4 sin 2 x )n

for continuity in [0, ]. Plot its graph and state the nature of

discontinuity and jump of discontinuity if applicable

Extra Edge Assignment

Subject : Mathematics
2 1
2

Assignment 30

dx

1.

Evaluate :

2.

Integrate

3.

Find the range of the function f(x) = sin1 x2 + n x [ x ] + cot1

4.

Integrate

5.

Integrate

6.

(2x 1) x 2 x

(p

6p) sin p dp

1 2 x2

where {*} and [*] are fractional part function and greatest function respectively.
2 4 x dx
x1/ 4 5
dx
x 16


Let f : (0, ) , be defined as, f(x) = arc tan (n x)
2 2
(A) Prove that f is invertible
(B) If g is the inverse of f, find g(/4)

(C) Sketch the graph of f(x),

2 x
e2

(D) evaluate :

f (x)
dx
x

x
sin dx
2 4

7.

Evaluate :

8.

If (sin x + cos x)2 + k sin x = 1 holds x R then find the value of k.

9.
10.

x + sin (32 + x) 18 cos (19 x) + cos (56 + x) 9 sin (x


+ sin
If the expression cos x
2
2

+ 17) is expressed in the form of a sinx + b cosx find the value of a + b

3 statements are given below each of which is either True or False. State whether True of False with appropriate reasoning. Marks will be allotted only if appropriate reasoning is given.
.
(log3169) (log13243) = 10
.
cos (cos ) = cos (cos 0)
.

cos x +

1
3
=
cos x
2

3
1
1
+
cos 2t +
cos 4t.
8
8
2

11.

Prove the identity cos4t =

12.

Suppose that for some angles x and y the equations sin2x + cos2y =
hold simultaneously. Determine the possible values of a.

3a
and
2

cos2x + sin2y =

a2
2

Extra Edge Assignment

Subject : Mathematics
Assignment 31
1.
Find the sum of all the solutions of the equation (log27x3)2 = log27x6.

10 y 10 y
<x<
and y = log10(tan x + sec x). Then the expression E =
simplifies to one of the six
2
2
2
trigonometric functions. find the trigonometric function.

2.

If

3.

If log2(log2(log2x)) = 2, then find the number of digits in x. You may use log102 = 0.3010.

4.

Assuming that x and y are both + ve satisfying the equation log (x + y) = logx + log y find y in terms of x.
Base the logarithm is 10 everywhere.
cos x cos 3 x
.
sin 3 x sin x

5.

If x = 7.5 then find the value of

6.

Find the solutions of the equations log

7.

Given that loga2 (a2 + 1) = 16 find the value of loga32 a


a

8.

If cos =

2 sin x (1

4
find the values of
5

(i) cos 3

(ii)

tan

+ cos x) = 2 in the interval x [0, 2]

9.

If log1227 = a find the value of log616 in term of a.

10.

Prove the identity,

11.

Find the exact value of cos 24 cos 12 + cos 48 cos 84

12.

Solve the system of equations 5(logxy + logyx) = 26 and xy = 64

13.

( 2r 1)

sin

Prove that
8

r 1

14.

side only.

x
sin x cos x 1 1 sin x
=
= tan , wheneven it is defined. Starting with left hand
sin x cos x 1
cos x
4 2

r 4

( 2r 1)

cos
Also find their exact numerical value.
=
8

r 1
r 4

1
1

Solve for x : log2 (4 x) + log (4 x). log x 2 log2 x = 0


2
2

15.

Number of zeros of the cubic f(x) = x3 + 2x + k k R, is


(A) 0
(B) 1
(C) 2

16.

The value of Lim


x
(A) 0

d
dx

r3
(r 1)(r 1) dx. is

(B) 1

(C) 1/2

(D) 3

(D) non existent

Extra Edge Assignment

Subject : Mathematics
1.

There are two numbers x making the value of determinant


numbers, is
(A) 4

2.

3.

Assignment 32

(B) 5

2x

equal to 86. The sum of these two

(C) 3

(D) 9

A function f(x) takes a domain D onto a range R if for each y R, there is some x D for which f(x) = y.
Number of function that can be defined from the domain D = {1, 2, 3} onto the range R = {4, 5} is
(A) 5
(B) 6
(C) 7
(D) 8
Suppose f, f, and f are continuous on [0, e] and that f(e) = f(e) = f(1) = 1 and
of

f ( x) nx dx equals

x
e

f(x)

dx = 1 , then the value


2

(A)

1
5

e
2

(B)

1
3

e
2

(C)

1
1

e
2

(D) 1

1
e

4.

A circle with centre C(1, 1) passes through the origin and intersect the x-axis at A and y-axis at B. The area
of the part of the circle that lies in the first quadrant is
(A) + 2
(B) 2 1
(C) 2 2
(D) + 1

5.

The planes 2x 3y + z = 4 and x + 2y 5z = 11 intersect in a line L. Then a vector parallel to L, is

6.

A fair dice is thrown 3 tims. The probability that the product of the three outcomes is a prime number, is
(A) 1/24
(B) 1/36
(C) 1/32
(D) 1/8

7.

Period of the function, f(x) = [x] + [2x] + [3x] + ..... + [nx]

8.

(A) 13 i 11j 7k

(B) 13 i 11j 7k

(C) 13 i 11j 7k

where n N and [ ] denotes the greatest integer function, is


(A) 1
(B) n
(C) 1/n

Let Z be a complex number given by, Z =

2i
3

10

i
i
1

1
1
1

(D) i 2j 5k

n(n 1)
x
2
(D) non periodic

the statement which does not hold good, is

(A) Z is purely real


(B) Z is purely imaginary
(C) Z is not imaginary
(D) Z is complex with sum of its real and imaginary part equals to 10
9.

Let f(x, y) = xy2 if x and y satisfy x2 + y2 = 9, then the minimum value of f(x, y) is

10.

Lim

(A) 0
x 0

(A)

1 3x 1 x

(1 x )101 1 101x
3
5050

(B) 3 3

(C) 6 3

(D) 3 6

has the value equal to


(B)

1
5050

(C)

1
5051

(D)

1
4950

Extra Edge Assignment

Subject : Mathematics
1.

Assignment 33

Number of positive solution which satsify the equation


log2x . log4x . log6x = log2x . log4 + log2x . log6x + log4x . log6x ?
(A) 0
(B) 1
(C) 2
(D) infinite

2.

Number of real solution of the equation 16 sin1x tan1x cosec1x = 3 is/are


(A) 0
(B) 1
(C) 2
(D) infinite

3.

Length of the perpendicular from the centre of the ellipse 27x 2 + 9y2 = 243 on a tangent drawn to it which
makes equal to intercept on the coordinates axes is
(A)

4.

3
2

(B)

6.

(C) 3 2

(D) 6

1 x2
2x

1
where x (1, 0), then f simplifies to
Let f(x) = cos1
2 + tan
1 x2
1 x
(B) /4

(A) 0
5.

(C) /2

(D)

A person throws four standard six sided distinguishable dice. Number of ways in which he can throw if the
product of the four number shown on the upper faces is 144 is
(A) 24
(B) 36
(C) 42
(D) 48

4 x
c

q r and suppose that det. (A) = 2 then the det. (B) equals, where B = 4 y
4z
y z
(A) det (B) = 2
(B) det (B) = 8
(C) det (B) = 16
(D) det (B) = 8

Let A = p
x

7.

The digit at the unit place of the number (2003)2003 is


(A) 1
(B) 3
(C) 7

8.

Let ABCDEFGHIJKL be a regular dodecagon, then the value of


(A) 4

(B) 2 3

(C) 2 2

2a
2b
2c

q
r

(D) 9

AB
AF
+
is
AF
AB

(D) 2

9.

Urn A contain 9 red balls and 11 white balls. Urn b contains 12 red balls and 3 white balls. One is to roll a
single fair die. If the result is a one or a two, then one is randomly select a ball from urn A. Otherwise one is
to randomly select a ball form urn B. The probability of obtaining a red ball, is
(A) 41/60
(B) 19/60
(C) 21/35
(D) 35/60

10.

Let f be a real valued function of real and positive argument such that
f(x) + 3x f(1/x) = 2(x + 1) for all real x > 0. The value of f(10099) is
(A) 550
(B) 505
(C) 5050
(D) 10010

11.
12.



+
is equal to
If and be the roots of the equation x + 3x + 1 = 0 then the value of
1
1
(A) 15
(B) 18
(C) 21
(D) none
2

The equation (x 1) (x 2) (x 3) = 24 has the real root equal to 'a' and the complex roots b and c. Then the
value of bc/a, is
(A) 1/5
(B) 1/5
(C) 6/5
(D) 6/5

Extra Edge Assignment

Subject : Mathematics
1.

2.
3.

Assignment 34

If m and n are positive integers satisfying 1 + cos 2 + cos 4 + cos 6 + cos 8 + cos 10 =
then m + n is equal to
(A) 9

(B) 10

(C) 11

cos m. sin n
,
sin

(D) 12

A circle of radius 320 units is tangent to the circle of radius 1000. The smaller circle is tangent to a diameter
of the larger circle at the point P. Least distance of the point P from the circumference of the larger circle is
(A) 300
(B) 360
(C) 400
(D) 420
In which of the following cases limit exists at the indicated points.

xe1 / x
[ x | x |]
at x = 0
(B) f(x) =
at x = 0
x
1 e1/ x
where [x] denotes the greatest integer functions.

(A) f(x) =

(C) f(x) = (x 3)1/5 sgn (x 3) at x = 3

(D) f(x) =

where sgn stands for signum function.

tan 1 | x |
at x = 0
x

4.

Let A and B are two independent events. If P(A) = 0.3 and P(B) = 0.6, then
(A) P(A and B) = 0.18
(B) P(A) is equal to P(A/B)
(C) P(A or B) = 0
(D) P(A or B) = 0.72

5.

Let T be the triangle with vertices (0, 0), (0, c2) and (c, c2) and let R be the region between y = cx and y = x2
where c > 0 then
(A) Area (R) =

c3
6

(B) Area of R =

c3
3

x 3
n

x 1

(C) cLim
0

Area(T )
Area(T )
3
= 3 (D) cLim
=
0 Area(R )
Area(R)
2

6.

Consider the graph of the function f(x) = e


(A) range of the function is (1, )
(C) graph lies completely above the x-axis

7.

Let f1(x) = x, f2(x) = 1 x ; f3(x) =

8.

The graph of the parabolas y = (x 2)2 1 and y = (x 2)2 1 are shwon.


Use these graphs to decide which of the statements below are true.
(A) Both function have the same doamin
(B) Both functions have the same range
(C) Both graphs have the same vertex.
(D) Both graphs have the same y-intercepts.

9.

10.

. Then which of the following is correct.


(B) f(x) has no zeroes
(D) domain of f is ( , 3) (1, )

1
1
x
x 1
, f4(x) =
; f5(x) =
; f6(x) =
x
1 x
x 1
x
Suppose that f6(fm(x)) = f4(x) and fn(f4(x)) = f3(x), then
(A) m = 5
(B) n = 5
(C) m = 6
(D) n = 6
y

ax 1
where a2 + b2 0, then Lim f(x)
Consider the function f(x) =
x
bx 2
(A) exists for all values of a and b
(B) is zero for a < b
(C) is non existent for a > b
(D) is e(1/a) or e (1/b) if a = b
x

Which of the following functions would represent a non singular mapping


(A) f : R R
f(x) = |x| Sgn x
(B) g : R R
where sgn denotes signum function

g(x) = x3/5

(C) h : R R

k(x) =

h(x) = x4 + 3x2 + 1

(d) k : R R

3x 2 7 x 6
x x2 2

Extra Edge Assignment

Subject : Mathematics

1.

2.

3.

Match the column

Column
(A)
Constant function f(x) = c, c R

Column
(P)
Bound

(B)

The function g(x)

(Q)

periodic

(C)
(D)

The function h(x) = arc tan x is


The function k(x) = arc cot x is

(R)
(S)

Monotonic
neither odd nor even

Column
(A)
cot1 (tan (37))
(B)
cos1(cos (233))

Column
(p)
143
(q)
127

(C)

(r)

3
4

(s)

2
3

(q)

Suppose sin cos = 1, then the value of sin3 cos3 (r)


is ( R)

t ( x 0) , is
x

dt

1
1 1
sin 2 cos 9

1
1
cos 2 arec cos

Column
(A)
Number of integral values of x satisfying the
(D)

inequality
(B)

(C)
(D)
4.

Assignment 35

x 1
x2

x 3
x4

The quadratic equations 2006x2 + 2007 x + 1 = 0


and x2 + 2007x + 2006 = 0 have a root in common.
Then the product of the uncommon roots is

The value of the limit, Lim sin 2x 2 tan x is


x 0
n(1 x 3 )

Column
(p)
1

(s)

11

A quadratic polynomial f(x) = x2 + ax + b is formed with one of its zeros being

43 3
2 3

where a and b are

43 3
=
integers. Also g(x) = x4 + 2x3 10x2 + 4x 10 is a biquadratic polynomial such that g
2 3 c 3 d

where c and d are also integers


Column
(A)
a is equal to
(B)
b is equal to
(C)
c is equal to
(D)
d is equal to

Column
(p)
4
(q)
2
(r)
1
(s)
11

5.

Let y = sin1(sin 8) tan1(tan 10) + cos1(cos 12) sec1(sec 9) + cot1(cot 6) cosec1(cosec 7). If y
simplifies to a + b, then find (a b)

6.

Suppose a cubic polynomial f(x) = x3 + px2 + qx = 72 is divisible by both x2 + ax + b and x2 + bx + a (where


a, b, p, q are constants and a b). Find the sum of the squares of the roots of the cubic polynomial

Extra Edge Assignment

Subject : Mathematics
1.

Let F(x) =
derivative.

4 t 2 dt and G(x) =

Assignment 36

4 t 2 dt then compute the value of (FG) (0) where dash denotes the

2.

10 identical balls are to be distributed in 5 different boxes kept in row and labelled A, B, C, D and E.
Find the number of ways in which the balls can be distributed in the boxes if no two adjacent boxes remain
empty.

3.

If f (x) = 4x2 + ax + (a 3) is negative for atleast one negative x, find all possible values of a.

4.

Let f(x) = sin6x + cos6x + k(sin4x + cos4x) for some real number k. Determine
(A) all real numbers k for which f(x) is constant for all values of x.
(B) all real numbers k for which there exists a real number 'c' such that f(c) = 0
(C) If k = 0.7, determine all solution to the equation f(x) = 0

5.

Let x0 = 2 cos

and xn =
6

find Lim
2(n + 1) .
n
6.

7.

Let f(x) =

8 sin

2 xn .

3
2005
1
2
+ ........ + f

+ f
+f
, then find the value of the sum f
2006
2006
9 3
2006
2006
9x

x 2 sin x

2 x n1 , n = 1, 2, 3, ........

dx

(a

1/ a

4 x a 5 x 2 )e ax dx

8.

For a > 0, find the minimum value of integral

9.

4
3
The set of real values of 'x' satisfying the equality + = 5 (where [ ] denotes the greatest integer
x
x

function) belongs to the interval a ,

a + b + c + abc.

b
b
where a, b, c N and
is in its lowest form. Find the value of
c
c

3
x 2 nt dt , find d y at x = e
dx 3
1

10.

If y =

11.

Find the equation of the normal to the curve y = (1 + x)y + sin1(sin2x) at x = 0.

12.

Find the real number 'a' such that 6 +

13.

The tangent to y = ax2 + bx +

f ( t ) dt
t2

= 2 x .

7
at (1, 2) is parallel to the normal at the point (2, 2) on the curve y = x 2 + 6x
2
+ 10. Find the value of a and b.

Extra Edge Assignment

Subject : Mathematics
Assignment 37
1.
Let f be a real valued function satisfying f(x) + f(x + 4) = f(x + 2) + f(x + 6), then prove that the function g(x) =

f (t) dt is a constant function.

x 8
x

2.
3.
4.

5.

A tangent drawn to the curve C1 y = x2 + 4x + 8 at its point P touches the curve C2 y = x2 + 8x + 4 at its
point Q. Find the coordinatest of the point P and Q, on the curves C1 and C2.

Given real numbers a and r, consider the following 20 numbers : ar , ar2, ar3, ar4, ....., ar20. If the sum of the 20
numbers is 2006 and the sum of the reciprocal of the 20 number is 1003, find the product of the 20 numbers.
Let f(x) and g(x) are differentiable functions satisfying the conditions
(i)
f(0) = 2 ; g(0) = 1
(ii)
f(x) = g(x)
and
Find the functions f(x) and g(x)
3 (b 3 b 2 b 1)
x
(b 2 3b 2)
Let f(x) =
2x 3

,
,

0 x 1

1 x 3

(iii) g(x) = f(x)

Find all possible real values of b such that f(x) has the smallest value at x = 1.
6.

There is a function f defined and continuous for all real x, which satisfies an equationof the form

f ( t ) dt =

1
x

t 2 f ( t ) dt +

value of the constant

x16
x18
+
+ C, where C is a constant. Find an explicit formula for f(x) and also the
8
9

f ( tx ) dt = nf(x) , then find f(x) where x > 0.


1

7.

Given

8.

Tangent at a point P1 [other than (0, 0)] on the curve y = x3 meets the curve again at P2. The tangent at P2
meets the curve at P3 and so on. Show that the abscissae of P1, P2, P3, ......, Pn, form a G.P. Also find the

area(P1P2P3 )
ratio area(P P P )
2 3 4
9.

The sum of the first five terms of a geometric series is 189, the sum of the first six terms is 381, and the sum
of the first seven terms is 765. What is the common ratio in this series.

10.

Form a quadratic equation with rational coefficients if one of its root is cot2 18

11.

Let and be the roots of the quadratic equation (x 2) (x 3) + (x 3)(x + 1) + (x + 1) (x 2) = 0. Find the
value of

12.
13.

1
1
1
+
+
.
( 1)( 1)
( 2)( 2)
( 3)( 3)

If a sin2x + b lies in the interval [2, 8] for every x R then find the value of (a b).

For x 0, what is the smallest possible value of the expression log(x3 4x2 + x + 26) log(x + 2) ?

14.

The coefficients of the equation ax2 + bx + c = 0 where a 0, satisfy the inequality


(a + b + c) (4a 2b + c) < 0 . Prove that this equation has 2 distinct real solutions.

15.

In an arithmetic progression, the third term is 15 and the eleventh term is 55. An infinite geometric
progression can be formed beginning with the eight term of this A.P. and followed by the fourth and second
term. Find the sum of this geometric progression upto n terms. Also compute S if it exists.

Extra Edge Assignment

Subject : Mathematics
Assignment 38
1.
Find the solution set of this equation log|sin x| (x2 8x + 23) > log|sin x|(8) in x [0, 2].

= ( p q ) ( r s) .
24

2.

Find the positive integers p, q, r satisfying tan

3.

Find the sum of n terms of the series .

4.

If sinx, sin22x and cos x. sin 4x form an increasing geometreic sequence, find the numerical value of cos 2x.
Also find the common ratio of geometric sequence.

Also find the sum if it exist if n

3
4
5
1
2
+
+ +
+
+ .........
8
16
32
2
4

5.

Find all possible parameters 'a' for which f(x) = (a2 + a 2) x2 (a + 5) x 2 is non positive for every x [0,1].

6.

The 1st, 2nd and 3rd terms of an arithmetic series are a, b and a2 where 'a' is negative. The 1st, 2nd and 3rd terms
of a geometric series are a, a2 and b find the
(a)
value of a and b
(b)
sum of infinite geometric series if it exists. If no then find the sum of n terms of the G.P.
(c)
sum of the 40 term of the arithmetic series.

7.

The nth term, an of a sequence of numbers is given by the formula an = an 1 + 2n for n 2 and a1 = 1. Find an
equation expressing an as a polynomial in n. Also find the sum to n terms of the sequence.

8.

Let f(x) denote the sum of the infinite trigonometric series, f(x) =

sin 3

n 1

2x
n

sin

3n

Find f(x) (independent of n) also evaluate the sum of the solutions of the equation f(x) = 0 lying in the interval
(0, 629)

(24 2x x
b

2 1/ 2

9.

Find the value of a and b where a < b, for which the integral

10.

1
sin x cos x
y = x
Solve the differential equation : y + x
e cos x
e cos x

11.

Integrate :

12.

In a ABC, given sin A : sin B : sin C = 4 : 5 : 6 and cosA : cos B : cos C = x : y : z. Find the ordered pair
that (x, y) that satisfies this extended proportion

13.

14.

x
1

sin 1 x
2

x 1

has the largest value.

x2
dx
( x cos x sin x )( x sin x cos x )

dx

Find the general solution of the equation, 2 + tan x . cot

x
x
+ cot x . tan
=0
2
2

Extra Edge Assignment

Subject : Mathematics
1.

Assignment 39

Let , be the distinct positive roots of the equation tan x = 2x then evaluate
independent of and .

(sin x. sin x ) dx ,
1

2.

Find the set of value of 'a' for which the quadratic polynomial
(a + 4)x2 2ax + 2a 6 < 0 x R.

3.

Solve the inequality by using method of interval,

4.

Find the minimum vertical distance between the graphs of y = 2 + sinx and y = cosx.

5.

Solve

6.

If p, q are the roots of the quadratic equation x 2 + 2bx + c = 0 , prove that

7.

Find the maximum and minimum value of y =

8.

Suppose that a and b are positive real numbers such that log27a + log9b = 7/2 and log27b + log9a = 2/3. Find
the value of the ab.

9.

Given sin2y = sinx . sin z where x, y, z are in an A.P.. Find all possible values of the common difference of the
A.P. and evaluate the sum of all the common differences which lie in the interval (0, 315).

10.

Prove that

11.

Find the exact value of tan2

12.

Find the value of k for which one root of the equation of x 2 (k + 1) x + k2 + k 8 = 0 exceed 2 and other is
smaller than 2.

13.

Let an be the nth term of an arithmetic progression. Let Sn be the sum of the first n terms of the arithmetic
progression with a1 = 1 and a3 = 3a8. Find the largest possible value of Sn.

14.

(a)

x 1 x 5

x 1
x 1

d 3
cos x cos 3 x when x = 18
dx 4

2 log y p y q = log 2 + log y b y 2by c .

(b)

x 2 14 x 9
x 2 2x 3

xR

tan 8
= (1 + sec 2) (1 + sec 4) (1 + sec 8).
tan

tan 8
3
5
7
+ tan2
+ tan2
+ tan2
.
tan
16
16
16

C
A
B
If A + B + C = and sin A = k sin
, then find the value of tan
tan
an
in terms of k.
2

2
2
2

Solve the inequality, log0.5

log6 x x

x 4 < 0.

15.

Given the product p of sines of the angles of a triangle and product q of cosines, find the cubic equation,
whose coefficeint are functions of p & q & whose roots are the tangents of the angles of the triangle.

16.

Evaluate :

1 (tan n
89

n 1

o 2

Extra Edge Assignment

Subject : Mathematics

Assignment 40

1.

Let f(x) = 1 x x . Find all real values of x satisfying the inequality , 1 f(x) f 3(x) > f(1 5x)

2.

Integrate :

3.

The circle C : x2 + y2 + kx + (1 + k) y (k + 1) = 0 passes through the same two points for every real number
k. Find
(i)
the coordinates of these two points
(ii)
the minimum value of the radius of a circle C.

4.

Comment upon the nature of roots of the quadratic equation x + 2x = k + | t k | dt depending on the value

(e

e2x e x 1

sin x cos x )(e x cos x sin x )

dx

of k R.
5.
6.

7.

3n Cn

Lim
Given n 2n

Cn

1/ n

a
where a and b are relatively prime, find the value of (a + b).
b

If each pair of the equations


x2 + p1x + q1 = 0
x2 + p2x + q2 = 0
x2 + p3x + q3 = 0
has exactly one root in common then show that (p1 + p2 + p3)2 = 4(p1p2 + p2p3 + p3p1 q1 q2 q3)
Let a, b, c be three sides of a triangle. Suppose a and b are the roots of the equation
x2 (c + 4) x + 4(c + 2) = 0 and the largest angle of the triangle is degrees. Find .

8.

Find the value of the definite integral

9.

Let tan . tan =

10.

1 5
2

x2 1

x4 x2 1

2005

2 sin x 2 cos x dx

. Find the value of (1003 1002 cos 2) (1003 1002 cos 2)

n 1 x dx
x

11.

Two vectors e1 and e 2 with | e1 | 2 and | e 2 | 1 and angle between e1 and e 2 is 60. The angle

between 2t e1 7e 2 and e1 te 2 belongs to the interval (90, 180). Find the range of t.

12.

A function f(x) continuous on R and periodic with period 2 satisfies f(x) + sinx . f(x + ) = sin2x
Find f(x) and evaluate :

f (x)dx

Extra Edge Assignment

Subject : Mathematics
1.

2.

3.

4.

5.

Lim
x 1

sin2 ( x 3 x 2 x 3)
1 cos( x 2 4x 3)

(A) 18

Let f(x) =

dt

(A) 1/11

has the value equal to

(B) 9/2

t 4 3t 2 13

Assignment 41

(C) 9

(D) none

. If g(x) is the inverse of f(x), then g(0) has the value equal to

(B) 11

(C) 13

(D)

13
The function f(x) has the property that for each real number x in its domain, 1/x is also in its domain and
f(x) + f(1/x) = x. The largest set of real numbers that can be in the domain of f(x), is
(A) {x | x 0}
(B) {x | x > 0}
(C) {x | x 1 and x 0 and x 1}
(D) {1, 1}
Let w =

z 2 3z 6
and z = 1 + i, then |w| and amp w respectively are
z 1

(A) 2, /4
If

(B)

1 cos a tan 2 (a / 2)
sin (a / 2)
2

2 , /4

(C) 2, 3/4

2 , 3/4

k cos a
where k, w and p have no common factor other than 1, then the value
w p cos a

of k + w2 + p2 is equal to
(A) 3
(B) 4
2

(D)

(C) 5

(D) 6

6.

In a birthday party, each man shook hands with everyone except his spouse, and no handshakes took place
between women. If 13 married couples attended, how many handshakes were there among these 26 people?
(A) 185
(B) 234
(C) 312
(D) 325

7.

If x and y are real numbers such that x2 + y2 = 8, then maximum possible value of x y is

8.

Let u(x) and v(x) are differentiable fundtions such that

9.

10.

(A) 2

has the value equal to


(A) 1

(C)

(B) 0

2

The coefficient of x when x

(A) 30C14 . 29

30

(D) 4

u( x )
u( x )
= 7. If
= p and
v( x )
v( x )

(C) 7

pq
u( x )

= q, then
pq
v( x )

(D) 7

is expanded and simplified is

(B) 30C16. 214

(C) 30C9 . 221

(D) 10C9

Let C be the circle described by (x a)2 + y2 = r2 where 0 < r < a. Let m be the slope of the line through the
origin that is tangent to C to a point in the first quadrant. Then
(A) m =

11.

(B)

a r
2

(B) m =

a2 r 2
r

(C) m =

r
a

(D) m =

What can one say about the local extrema of the the function f(x) = x + (1/x) ?
(A) The local maximum of f(x) is greater than the local minimum of f(x)
(B) The local minimum of f(x) greater than the local maximum of f(x) .
(C) The function f(x) does not have any local extrema.
(D) f(x) has one asymptote

a
r

Extra Edge Assignment

Subject : Mathematics
1.

2.

Assignment 42

If the function f(x) = 4x2 4x tan2 has the minimum value equal to 4 then the most general values of ''
given by
(A) 2n + /3
(B) 2n /3
(C) n /3
(D) 2n/3
where n

2
tan arc tan
arc tan( 5) equals
3

(A) 3

(B) 1

(C) 1

(D)

3.

A line passes through (2, 2) and cuts a triangle of area 9 square units from the first quadrant. The sum of all
possible values for the slope of such a line, is
(A) 2.5
(B) 2
(C) 1.5
(D) 1

4.

Which of the following statement is/are true concerning the general cubic
f(x) = ax3 + bx2 + cx + d (a 0 and a, b, c, d R)
.
The cubic always has atleast one real root
.
The cubic always has exactly one point of inflection
(A) Only
(B) only
(C) Both and are true (D) neither nor is true

5.

If S = 12 + 32 + 52 + ..... + (99)2 then the value of the sum 22 + 42 + 62 + ...... + (100) 2 is


(A) S + 2550
(B) 2S
(C) 4S
(D) S + 5050

6.

Through the focus of the parabola y2 = 2px (p > 0) a line is drawn which intersects the curve at A(x1, y1) and

y1y 2
B(x2, y2). The ratio x x equals
1 2
(A) 2

(B) 1

n.3n

(C) some function of p

1
, then the range of x is (n N)
3

7.

If

8.

The area of the regions enclosed by the curves y = x2 and y =

9.

Suppose that the domain of the function f(x) is set D and the range is the set R, where D and R are the
subsets of real numbers. Consider the functions : f(2x), f(x + 2), 2f(x), f(x/2), f(x)/2 2. If m is the number of
functions listed above that must have the same domain as f and n is the number of functions that must have
the same range as f(x), then the ordered pair (m, n) is
(A) (1, 5)
(B) (2, 3)
(C) (3, 2)
(D) (3, 3)

10.

n( x 2) n.3

(A) [2, 5)
(A) 1/3

n 1

(C) 4

(B) (1, 5)
(B) 2/3

x 2 2mx 1
f : R R is defined as f(x) =
mx 1

(C) 1/6

for
for

If f(x) is one-one then m must lies in the interval


(A) ( , 0)
(B) ( , 0]
11.

(D) ( , )

(C) (1, 5)

x0

x0

| x | is

(D) 1

(C) (0, )

(D) [0, )

Let A = {x | x2 + (m + 1) x 2(m + 1) = 0, x R} ; B = {x | (m 1) x2 + mx + 1 = 0, x R}. Number of values


of m such that A B has exactly 3 distinct elements is
(A) 4
(B) 5
(C) 6
(D) 7

Extra Edge Assignment

Subject : Mathematics

Assignment 43

Direction Q. 1 to 3

Consider the function defined on [0, 1] R , f(x) =


1.

2.

f ( x) dx equals
0

Lim
t 0

t2

(B) sin (1) 1

f (x) dx equals
0

xe x
Let f(x) =
2
3
x x x

(B) 1/6

x0

x0

(D) sin (1)

(C) 1/12

(D) 1/24

then the correct statement is

5.

Suppose f is defined from R [1, 1] as f(x) =

6.

the value of the definite integral

(B) f is continuous but not differentiable at x = 0


(D) f is continuous but not differentiable at x = 0

x2 1

x2 1

where R is the set of real number. Then the statement

which does not hold is


(A) f is many one onto
(B) f increases for x > 0 and decrease for x < 0
(C) minimum value is not attained even through f is bounded
(D) the area included by the curve y = f(x) and the line y = 1 is sq. units

(A)

x n 3 cos x dx

x n 3 cos x dx, is

3 cos x

3 cos x

(C) zero

3 cos x
(B) 2 x n
dx
3 cos x

(D) 2

x 3 (1 x ) sin(1/ x 2 )
f : [0, 1] R is defined as f(x) =
0

(A) f is continuous but not derivable in [0, 1]


(C) f is bounded in [0, 1]

8.

(C) sin (1)

(A) f is continuous and differentiable for all x.


(C) f is continuous and differentiable for all x.

7.

if x 0 and f(0) = 0

(A) 1/3
4.

x2

The function f(x) is


(A) has a removable discontinuity at x = 0
(B) has non removable finite discontinuity at x = 0
(C) has a non removable infinite discontinuity at x = 0
(D) is continuous at x = 0

(A) 1 sin (1)


3.

sin x x cos x

if
if

x n 3 cos x dx

3 cos x

0 x 1
x0

, then

(B) f is differentiable in [0, 1]


(D) f is bounded in [0, 1]

Let 2 sin x + 3 cos y = 3 and 3 sin y + 2 cos x = 4 then


(A) x + y = (4n + 1) /2, n
(B) x + y (2n + 1) /2, n
(C) x and y can be the two non right angles of a 3-4-5 triangle with x > y.
(D) x and y can be the two non right angles of 3-4-5 triangle with y > x.

Extra Edge Assignment

Subject : Mathematics

1.

2.

3.

The equation cosec x + sec x = 2 2 has



(A) no solution in 0 ,
4

(C) no solution in ,
2 4


(B) a solution in ,
4 2

3
(D) a solution in ,
4

For the quadratic polynomial f(x) = 4x2 8kx k, the statements which hold good are
(A) there is only one integral k for which f(x) is non negative x R
(B) for k < 0 the number zero lies between the zeroes of the polynomial
1 4
(C) f(x) = 0 has two distinct solutions in (0, 1) for k ,
4 7
(D) Minimum value of k R is k (1 + 12k)

Match the column


Column

Column

(A)

f(x) = sin 1(sin x) + cos1(cos x)

(p)

(B)

g(x) = sin1 |x| + 2 tan1 |x|

(q)

is increasing x (0, 1)

(r)

period is 2

(C)
(D)

4.

Assignment 44

2x
h(x) = 2 sin1
1 x2
k(x) = cot(cot1x)

, x [0, 1]

Column
(A)

(B)

(s)

range is [0, ]

is decreasing x (0, 1)

Centre of the parallelopiped whose 3 coterminous edges


OA , OB and OC have position vectors a , b and c


respectively where O is the origin, is
OABC is a tetrahedron where O is the origin. Positions vectors of

Column

(p)
abc

(q)


abc
3

(r)


abc
4

(s)


abc
2

its angular points A, B and C are a , b and c respectively..


Segments joininig each vertex with the centroid of the opposite
face are concurrent at a point P whose p.v.'s are
(C)

Let ABC ba a triangle the position vectors of its angular points

are a , b and c respectively. If | a b | = | b c | = | c a | , then


the p.v. of the orthocentre of the triangle is
(D)


Let a , b , c be 3 mutually perpendicular vectors of the same

magnitude. If an unknown vector x satisfies the equation



a(( x b) a) + b (( x c ) b) + c (( x a ) c ) = 0

4.

If the value of limit Lim


n

cos
n

k 2

1 1

(k 1)k(k 1)(k 2)
120
is equal to k , find the value of k.
k(k 1)

Extra Edge Assignment

Subject : Mathematics

1.

Column
(A)

If

Assignment 45

= (a b)(b c) (c a) (a + b + c) , then

the solution of the equation

1
( x a )2

( x b)( x c )

Column

1
( x b )2

( x c )( x a )

1
( x c )2

( x a )( x b)

(p)

abc
3

= 0, is

(B)

3 ( x a)( x b)( x c ) x

, is
the value of limit, Lim
x

(q)

(C)

ax bx c x
Lim
x 0
3

(r)

abc
2

(D)

1/ x

equals

Let a, b, c are distinct reals satisfying a3 + b3 + c3 = 3abc. If the (s)


quadratic equation (a + b c) x2 + (b + c a) x + (c + a b) = 0
has equal roots then a root of the quadratic equation is

abc

2.

Let f(x) = (x + 1) (x + 2) (x + 3) (x + 4) + 5 where x [6, 6]. If the range of the function is [a, b] where
a, b N then find the value of (a + b).

3.

Let =

/4
0

( x 4 x 2 ) n (1 + tan x) dx. If the value of =

where k N, find k
4.

3 n2
k

Suppose f and g are two functions such that f, g : R R,


2

f(x) = n 1 1 x and g(x) = n

x 1 x2

, then find the value of x eg(x)

0
rs .2r.5 s where rs =
1
s 1
n

if
if

rs
.
rs

1
f + g(x) at x = 1
x

Will the sum hold if n .

5.

Evaluate :

6.

Find the general solution of the equation, 2 + tan x . cot

7.


Given that 3 sin x + 4 cos x = 5 where x 0 , . Find the value of 2 sinx + cos x + 4 tan x.
2

8.

Find the integral solution of the inequality

9.

In ABC, suppose AB = 5 cm, AC = 7cm, ABC =

10.

The sides of a triangle are n 1 , n and n + 1 and the are is n n . Determine n.

11.

With usual notions, prove that in a triangle ABC, r + r1 + r2 r3 = 4R cos C.

(a)
(b)

r 1

Find the length of the side BC.


Find the area of ABC.

log0.3 ( x 1)

2x x 2 8

x
x
+ cot x. tan
=0
2
2

Extra Edge Assignment

Subject : Mathematics
1.

Assignment 46

Find the general solution of the equation, sin x + cos x = 0. Also find the sum of all solutions in [0, 100].

2.

Find all negative values of 'a' which makes the quadratic inequality sin2x + a cos x + a2 1 + cos x true for
every x R.

3.

Solve for x,

4.

In a triangle ABC if a2 + b2 = 101 c2 , then find the value of

5.

Solve the equation for x, 5 2 5 2

6.

Evaluate the sum

7.

Suppose that P(x) is a quadratic polynomial such that P(0) = cos340, P(1) = (cos 40)(sin2 40) and
P(2) = 0. Find the value of P(3).

8.

If , m, n are 3 numbers in G.P. prove that the first term of an A.P. th, mth, nth terms are in H.P. is to the
common difference as (m + 1) to 1

9.

Let a, b, c, d, e, f R such that ad + be + cf =

2 log ( x
5log2 x 3 4
2

n 1

/ 2)

log

log5 (sin x )

n2

2x 2

5log2 x

= 15 2

cot C
cot A cot B

log15 (cos x )

use vectors or otherwise to prove that,

(a 2 b 2 c 2 )(d2 e 2 f 2 )

abc

a b c
2

def

d2 e 2 f 2

10.

Let equation x3 4x2 + 5x 1.9 = 0 has real roots r, s, t.Find the area of the triangle with sides r, s and t.

11.

Suppose x3 + ax2 + bx + c satisfies f(2) = 10 and takes the extreme value

12.

Let =

value of a, b, and c

nx

1 y
x

xy 1

dx and J =

where c, d R. Hence show that


13.

2
50
where x = . Find the
3
27

x
nx x xy 1
dy where y = xy. Show that . J = (x + d) (y + c)
1 y

d
dy
(J) = + J
.
dx
dx

Let ai, i = 1, 2, 3, 4, be real numbers such that a1 + a2 + a3 + a4 = 0. Show that for arbitrary real numbers bi,
i = 1, 2, 3 the equation a1 + b1x + 3a2x2 + b2x3 + 5a3x4 + b3x5 + 7a4x6 = 0
has at least one real root which lies on the interval 1 x 1.

x
3

x2 1

dx

14.

Evaluate :

15.

Let x, y R in the interval (0, 1) and x + y = 1. Find the minimum value of the expression xx + yy.

16.

x 3 3x 2 x 1

(1 sin x )(2 sin x )


dx
(1 sin x )(2 sin x )

Extra Edge Assignment

Subject : Mathematics

Assignment 47

1.

If tan , tan are the roots of x px + q = 0 and cot , cot are the roots of x2 rx + s = 0, then find the
value of rs in terms of p and q.

2.

Let P(x) = ax2 + bx + 8 is a quadratic polynomial. If the minimum value of P(x) is 6 when x = 2, find the values
of a and b.

3.

10 2n 1
Let P =

, then find log0.01(P).


n1

4.

Prove that identity

5.

Find the general solution set of the equation logtan x (2 + 4 cos2x) = 2.

6.

Find the value of

7.

(a)

8.


The equation cos2x sin x + a = 0 has roots when x 0 , find 'a'.
2

9.

A, B and C are distinct positive integers, less than or equal to 10. The arithmetic mean of A and B is 9. The

10.

Express cos 5x in terms of cos x and hence find general solution of the equation cos 5x = 16 cos 5x.

11.

If x is real and 4y2 + 4xy + x + 6 = 0, then find the complete set of values of x for which y is real.

sec 8 A 1
tan 8 A
=
.
sec 4 A 1
tan 2A

sin sin 3 sin 5 ...... sin17

when =
.
cos cos 3 cos 5 ....... cos 17
24

1
1
1
+
+
+ ......
4.7.10
7.10.13
1.4.7
Sum the following series upto n-terms 1 . 2 . 3 . 4 + 2 . 3 . 4 . 5 . 6 + .........

Sum the following series to infinity

(b)

geometric mean of A and C is 6 2 . Find the harmonic mean of B and C.

12.

Find the sum of all the integral solutions of the inequality 2 log3x 4 logx27 5.

13.

1 tan 1 tan 1 tan


sin sin sin 1

2
2
2

If + + = , show that
=
cos cos cos

1 tan 1 tan 1 tan


2
2
2

14.

(a)

15.

The area of the region of the plane bounded above by the graph of x2 + y2 + 6x + 8 = 0 and below by the graph
of y = |x + 3|, is

C
C
+ (a + b)2 sin2
.
2
2
In any ABC prove that a3 cos(B C) + b3 cos(C A) + c3 cos(A B) = 3 abc.
In any ABC prove that c2 = (a b)2 cos2

(b)

(A)

(B)

2
4

(C)

(D)

16.

Consider straight line ax + by = c where a, b, c R+ and a, b, c are distinct. This line meets the coordinate
axes P and Q respectively. If area of OPQ, 'O' being origin does not depend upon a, b and c, then
(A) a, b, c are in G.P.
(B) a, c, b are in G.P.
(C) a, b, c are in A.P.
(D) a, c, b are in A.P.

17.

If x and y are real numbers and x2 + y2 = 1, then the maximum value of (x + y)2 is
(A) 3

(B) 2

(C) 3/2

(D)

Extra Edge Assignment

Subject : Mathematics
1.

The value of the definite integral


(A) /4

2.

(B) /2

a2 b2 c 2
2bc

5.

6.

dx

(a > 0) is

)(1 x 2 )

(C)

(B)

c 2 a2 b2
2bc

(D) some function of a

cos

(C)

a
b
c
cos cos
r
r
r
equals
b
c
sin sin
r
r

b2 c 2 a2
2bc

(D) independent of a, b and c

a curve y = f(x) such that f(x) = 4x at each point (x, y) on it and crosses the x-axis at (2, 0) at an angle of
45. The value of f(1) is
(A) 5

4.

(1 x

Let a, b, c are non zero constant number, then Lim


r
(A)

3.

Assignment 48

(B) 15

(C)

sin x

The minimum value of the function f(x) =

55
3

(D)

cos x

1 cos x
1 sin x
varies over all numbers in the largest possbile domain of f(x) is
(A) 4
(B) 2
(C) 0
2

tan x

35
3

sec x 1
2

cot x

cos ec 2 x 1

as x

(D) 2

a non-zero polynomial with real coefficient has the property that f(x) = f(x) . f(x). The leading coefficient of
f(x) is
(A) 1/6
(B) 1/9
(C) 1/12
(D) 1/18
Let Cn =
(A) 1

1/ n
1
n 1

tan 1(nx )
sin 1(nx )

dx , then Lim n2.C equals


n
n

(B) 0

(C) 1

(D) 1/2

7.

Let z1, z2, z3 be complex numbers such that z1 + z2 + z3 = 0 and |z1| = |z2| = |z3|, then z12 + z22 + z32, is
(A) greater than zero
(B) equal to 3
(C) equal to zero
(D) equal to 1

8.

Number of rectangles with sides parallel to the coordinate axes whose vertices are all of the form (a, b) with
a and b integers such that 0 a, b n, is (n N)
(A)

n2 (n 1)2
4

(B)

(n 1)2 n2
4

(C)

(n 1)2
4

9.

Number of roots of the function f(x) =

10.

If p(x) = ax2 + bx + c leaves a remainder of 4 when divided by x, a remainder of 3 when divided by x + 1, and
a remainder of 1 when divided by x 1, p(2) is
(A) 3
(B) 6
(C) 3
(D) 6

11.

Let f(x) be a function that has a continuous derivative on [a, b] , f(a) and f(b) have opposite signs, and
f(x) 0 for all numbers x between a and b (a < x < b). Number of solution does the equation f(x) = 0 have
(a < x < b).
(A) 1
(B) 0
(C) 2
(D) cannot be determined

(A) 0

(B) 1

( x 1)

3x + sin x is

(D) n2

(C) 2

(D) more than 2

Extra Edge Assignment

Subject : Mathematics

1.

Assignment 49

Which of the following definite integral has a positive value ?


(A)

2 / 3
0

sin( 3 x ) dx

(B)

sin(3 x ) dx

2 / 3

(C)

sin(3 x ) dx

3 / 2

(D)

sin(3x ) dx

3 / 2
0

2.

Let set A consist of 5 elements and set B consists of 3 elements. Number of function s that can be defined
from A to B which are neither injective nor surjective, is
(A) 99
(B) 93
(C) 123
(D) none

3.

A circle with centre A and radius 7 is tangent to the sides of an


angle of 60. A larger circle with centre B is tangent to the sides of
the angle and to the first circle. The radius of the larger circle is
(A) 30 3

4.

5.

(B) 21

(C) 20 3

(D) 30


The value of the scalar (p q) . ( r s) can be expressed in the determinant form as







q. r
q. s
p. r
p.s
p. r
q. s
p. r



(A)
(B)
(C)
(D)
p. r
p.s
q. s q. r
p. r
p.s
q. r

If Lim
x. n
x

/x
0

1/ x
0

(A) = 2, = 1, R
(C) R, = 1, R

1/ x


p.s

q. s

= 5, where , , are finite real numbers then


(B) = 2, = 2, = 5
(D) R, = 1, = 5

6.

If f(x, y) = sin1 (|x| + |y|), then the area of the domain of f, is

7.

A , B and C are distinct positive integers, less than or equal to 10. The arithmetic mean of A and B is 9. The

(A) 2

(C) 4

(D) 1

geometric mean of A and C is 6 2 . The harmonic mean of B and C is


(A) 9

8.

(B) 2 2

9
19

(B) 8

8
9

(C) 2

7
19

(D) 2

8
17

If x is real and 4y2 + 4xy + x + 6 = 0, then the complete set of values of x for which y is real, is
(A) x 2 or x 3
(B) x 2 or x 3
(C) 3 x 2
(D) x 3 or x 2

9.

If alternatively toss a fair coin and throw a fair die until ,either toss a head or throw a 2. If toss the coin first,
the probability that throw a 2 before toss a head, is
(A) 1/7
(B) 7/12
(C) 5/12
(D) 5/7

10.

Let A, B, C, D be (not necessarily square) real matrices such that


AT = BCD ; BT = CDA ; CT = DAB and DT = ABC for the matrix S = ABCD, consider the two statements.

S3 = S
.
S2 = S4
(A) is true but not
(B) is true but not
(C) both and are true (D) both and are false

11.

the function f(x) is defined for x 0 and has its inverse g(x) which is differentiable. If f(x) satisfies
and g(0) = 0, then
(A) f(x) is an odd linear polynomial
(C) f(2) = 1

(B) f(x) is some quadratic polynomial


(D) g(2) = 4

f (t ) dt x

g( x )
0

Extra Edge Assignment

Subject : Mathematics

1.

Assignment 50

Consider a triangle ABC in xy plane with D, E and F as the middle points of the sides BC, CA and AB
respectively. If the coordinates of the points D, E and F are (3/2, 3/2) ; (7/2 , 0) and (0, 1/2) then which of
the following are correct ?
(A) circumcentre of the triangle ABC does note lie inside the triangle
(B) orthocentre, centroid, circumcentre and incentre of triangle DEF are collinear but of triangle ABC are non
collinear
(C) Equation of a line passes through the orthocentre of triangle ABC and perpendicular to its plane is

r 2( i j ) k
(D) distance between centroid and orthocentre of the triangle ABC is

2.

If a continuous function f(x) satisfies the relation

t f ( x t ) dt =

numbers x, then which of the following does not hold good ?


(A) f(0) = 1
3.

f (t) dt + sin x + cos x x 1, for all real


x

(C) f(0) = 2

(D)

f ( x ) dx e

Match the column


Column I
(A)
(B)
(C)

(D)

2.

(B) f(0) = 0

5 2
.
3

Lim nx
x
x

Lim e
x

nt
x

dt

e3

x 4 1

ex

Column II
is
2

is

Lim (1)n sin n 2 0.5n 1 sin (n 1)


x

4n
is where n N
The value of the integral

x
tan 1

x 1
dx is
2
1

2
x

2
x
1

tan

4
a
and B =
1
0
T K
T
T K
R = P Q P also S = PBP and T = P S P
Column
3
Consider the matrices A =
1

(p)

(q)

1
2

(r)

(s)

non existent

b
and let P be any orthogonal matrix and Q = PAP
APT and
1

Column

(A)

If vw vary K from 1 to n thern the first row


first column elements at R will form

(p)

GP with common ratio a

(B)

If we vary K from 1 to n then the 2nd row 2nd


column elements at R will form

(q)

A.P. with common difference 2

(C)

If we vary K from 1 to n then the first row first


column elements of T will form

(r)

G.P. with common ratio b

(D)

If we vary K from 3 to n then the first row 2nd column


elements of T will represent the sum of P

(s)

A.P. with common difference 2

Extra Edge Assignment

Subject : Mathematics

1.

Assignment 51

Column I
(A)

Given two vectors a and b such that | a | | b | = | a b | = 1


The angle between the vectors 2a b and a is

Column II
(p)

30

(B)

In a scalene triangle ABC, if a cos A = b cos B, then C equals

(q)

45

(C)

In a triangle ABC, BC = 1 and AC = 2. The maximum possible


value which the A can have is

(r)

60

(s)

90

(D)
Subjective

In a ABC , B = 75 and BC = 2AD where AD is the


altitude from A, then C equals

x sin

/2

96 V
1
dx, find the value of

2.

Suppose V =

3.

One of the roots of the equation 2000 x6 + 100x5 + 10x3 + x 2 = 0 is of the form

4.

A circle C is tangent to the x and y axis in the first quadrant at the points P and Q respectively. BC and AD
are parallel tangents to the circle with slope 1. If the points A and B are on the y-axis while C and D are on

m n
, where m is non
r
zero integer and n and r are relatively prime natural numbers. Find the value of m + n + r.

the x-axis and the area of the figure ABCD is 900 2 square units, then find the radius of the circle
5.

6.
7.

8.

9.

10.

11.

Let f(x) = ax2 4ax + b (a > 0) be defined in 1 x 5. Suppose the average of the maximum value and the
minimum value of function is 14, and the difference between the maximum value and minimum value is
18.Find the value of a2 + b2 .
1
If the Lim
x 0 x 3

1
1 ax

exists and has the value equal to , then the value of 1 2 + 3 .

bx
a
b
1 x

8 clay targets have been arranged in vertical column, 3 being in the first column, 2 in second, and 3 in the
third. In how many ways can they be shot (one at a time) if no target below it has beent shot.
Evaluate :

x(sin (sin x) cos (cos x )) dx

Evaluate :

x(sin(cos

/2
0

x ) cos(sin 2 x )) dx

dx
x sin x cos x

Prove that

3n 1 3n 2 = 3 3

n 0

Extra Edge Assignment

Subject : Mathematics

1.

Assignment 52

If cos A, cos B and cos C are the roots of the cubic x3 + ax2 + bx + c = 0 where A, B, C are the angles of a
triangle then find the value of a2 2b 2c.

2.

Find all functions, f : R R satisfying (x f(x) 2F(x)) (F(x) x2) = 0 x R where f(x) = F(x).

3.

x 1

3x
3/2

4.

1/ 2

For a > 0, b > 0 verify that


evaluate :

5.

6.

nx

2x 4

nx

bx a

dx reduces to zero by a substitution x = 1/t. Using this or otherwise

dx

tan 1 x

dx
x

Find the sum of the infinite series


(A)

7.

ax

1
3

(B)

1
4

1
1
1
1
1
+
+
+
+
+ .......
9
18
30
45
63

(C)

1
5

Number of degrees in the smallest positive angle x such that


8 sin x cos5 x 8 sin5x cos x = 1, is
(A) 5
(B) 7.5
(C) 10

(D)

2
3

(D) 15

8.

There exist positive integers A, B and C with no common factors greater than 1, such that
A log2005 + B log2002 = C. The sum A + B + C equals
(A) 5
(B) 6
(C) 7
(D) 8

9.

A triangle with sides 5, 12 and 13 has both inscribed and circumscribed circles. The distance between the
centres of these circles is

10.

11.

5
(C) 65
(D)
2
The graph of a certain cubic polynomial is as shown. If the polynomial can
be written in the form f(x) = x3 + ax2 + bx + c, then
(A) c = 0
(B) c < 0
(C) c > 0
(D) c = 1
(A) 2

(B)

65
2

The sides of a triangle are 6 and 8 and the angle between these sides varies such that 0 < < 90. The
length of 3rd side x is
(A) 2 < x < 14
(B) 0 < x < 10
(C) 2 < x < 10
(D) 0 < x < 14

12.

The sequence a1 , a2, a3, ........ satisfies a1 = 19, a9 = 99, and for all n 3, an is the arithmetic mean of the first
n 1 terms. Then a2 is equal to
(A) 179
(B) 99
(C) 79
(D) 59

13.

If b is the arithmetic mean between a and x ; b is the geometric mean between 'a' and y ; 'b' is the harmonic
mean between a and z (a, b, x, y, z > 0), then the value of xyz is
(A) a3

(B) b3

(C)

b 3 (2a b)
2b a

(D)

b 3 (2b a)
2a b

Extra Edge Assignment

Subject : Mathematics

1.

Assignment 53

Given A(0, 0), ABCD is a rhombus of side 5 units where the slope of AB is 2 and the slope of AD is 1/2. The
sum ovf abscissa and ordinate of the point C is
(A) 4 5

(B) 5 5

(C) 6 5

(D) 8 5

2.

A cricle of finite radius with points (2, 2), (1, 4) and (k, 2006) can exist for
(A) no value of k
(B) exactly one value of k
(C) exactly two values of k
(D) infinite values of k

3.

If a, b, c are in G.P., then

4.

How many terms are there in the G.P. 5, 20, 80, ....... 20480.
(A) 6
(B) 5
(C) 7

5.

The sum of the first 14 terms of the sequence

(A) A.P.

(A)
6.

7(2 11 x )
1 x

1
1
1
,
,
are in
b a 2b b c
(B) G.P.
(C) H.P.

(B)

1 x
(C)

(D) 8

1
1
+
+ ..... is
1 x
1 x
14

(1 x )(1 x )(1 x )

(D) none

10
and xy = 144, then arithmetic mean of x and y is
3

If x, y > 0, logyx + logxy =


(A) 24

7(1 7 x )
1 x

(D) none

(B) 36

(C) 12 2

(D) 13 3

7.

A circle of radius R is circumscribed about a right triangle ABC. If r is the radius of incircle inscribed in
triangle then the area of the triangle is
(A) r(2r + R)
(B) r(r + 2R)
(C) R(r + 2R)
(D) R(2r + R)

8.

The simplest form of 1 +

(A) a for a 1
(C) a for a 0 and a 1
9.

10.

1
1 a

is
(B) a for a 0 and a 1
(D) 1 for a 1

If the quadratic equation ax2 + bx + c = 0 (a > 0) has sec2 and cosec2 as its roots then which of the
following must hold good ?
(A) b + c = 0
(B) b2 4ac 0
(C) c 4a
(D) 4a + b 0
Which of the following equations can have sec2 and cosec2 as its roots ( R)
(A) x2 3x + 3 = 0
(B) x2 6x + 6 = 0
(C) x2 9x + 9 = 0
(D) x2 2X + 2 = 0

11.

If a ABC is formed by 3 straight lines


u = 2x + y 3 = 0 ; v = x y = 0 and w = x 2 = 0 , then for k = 1 the line u + kv = 0 passes through its
(A) incentre
(B) centroid
(C) orthocentre
(D) circumcentre

12.

If a, b and c are numbers for which the equation


a + b + c equals
(A) 2

(B) 3

x 2 10 x 36
x( x 3)

(C) 10

c
b
a
+
+
is an identity, then
( x 3) 2
x 3
x
(D) 8

Extra Edge Assignment

Subject : Mathematics

1.
2.

3.

The equation | x 2 |10 x


(A ) 3 integral solutions
(C) 1 prime solution

3x
= | x 2 | has

(B) 4 real solutions


(D) no irrational solution

18

3 sec 18 is a rational which is not integral

(B)

The value of cosec

(C)

If x lies in the third quadrant, then the expression


independent of x.

x
2

4 sin 4 x sin 2 2x + 4 cos 4 2 is

(D)
there are exactly 2 values of in [0, 2] which satisfy 4 cos2 2 2 cos 1 = 0.
Column
Column
(p)

(q)

3/4

(r)

The equation ax2 + 3xy 2y2 5x + 5y + c = 0


represents two straight lines perpendicular to each
other, then |a + c| equals

(s)

3/2

(A)

In a triangle ABC, AB = 2 3 , BC = 2 6 , AC > 6

(p)

60

(B)

In a triangle ABC is b =

(q)

90

(C)

In a ABC if (a + b + c) (b + c a) = 3bc, then A equals

(r)

120

(B)
(C)
(D)

Area of the triangle formed by the straight lines


x + 2y 5 = 0, 2x + y 7 = 0 and x y + 1 = 0
in square units is equal to

Abscissa of the orthocentre of the triangle whose vertices


are the point (2, 1) ; (6, 1) and (2, 5)

Variable line 3x( + 1) + 4y( 1) 3( 1) = 0


for different values of are concurrend at the point (a, b)
The sum (a + b) is

Column

(D)
5.

Assignment 54

Which of the follwoing statements hold good ?


(A)
If M is the maximum and m is the minimum value of y = 3 sin2x + 3 sin x . cos x + 7 cos2x then the
mean of M and m is 5.

(A)

4.

and area of the triangle ABC is 3 6 . B equals


angle B equals

3 , c = 1 and A = 30, then

Area of a triangle ABC is 6 sq. units. If the radii of its


excircle are 2, 3 and 6 then largest angle of the triangle is

(s)

The sides of right traingle from a three term geometric sequence


The shortest side has length 2. The length of the hypotenuse

(q)

20

(r)

30

The diagonals of a parallelogram have a measure of 4 and 6


(s)
metres. They cut off forming an angle of 60. If the perimeter of the

40

Column
(A)
The sequence a, b, 10, c, d is an arithmetic progression.
The value of a + b + c + d
(B)

is of the form a +

(C)
(D)

Column

a + b equals
2

b where a N and

75

Column
(p)
10

b is a surd, then

The sum of first three consecutive numbers of an infinite G.P.


is 70, if the two extremes be multipled each by 4, and the mean
by 5, the products are in A.P. The first term of the G.P. is

parallelogram is 2( a b ) where a, b N, then (a + b) equals

Subject : Mathematics

Extra Edge Assignment

Assignment 55

1.

Consider the quadratic polynomial f(x) = x2 4ax + 5a2 6a.


(a)
Find the smallest positive integral value of 'a' for which f(x) is positive for every real x.
(b)
Find the largest distance between the roots of the equation f(x) = 0

2.

(a)
(b)

3.

Find the greatest value of c such that system of equations x 2 + y2 = 25, x + y = c has a real
solution
The equations to a pair of opposite sides of a parallelogram are
x2 7x + 6 = 0 and y2 14y + 40 = 0
find the equations to its diagonals.

Find the equation of the straight line with gradient 2 if it intercepts a chord of length 4 5 on the circle
x2 + y2 6x 10y + 9 = 0.
cos 3 2 x 3 cos 2x

4.

The value of the expression,

whereeven defined is independent of x. Without allotting a


cos 6 x sin6 x
particular value of x, find the value of this constant.

5.

Find the general solution of the equation sin3x (1 + cot x ) + cos3x( 1 + tanx) = cos 2x

6.

If the third and fourth terms of an arithmetic sequence are increased by 3 and 8 respectively, then the first four
terms form a geometric sequence. Find
(i)
the sum of the first four terms of A.P.
(ii)
second term of the G.P.

7.

(a)
(b)

1
or x = 15 satisfies the equation, log8(kx2 wx + f) = 2. If k, w and f are relatively prime
3
positive integers then find the value of k + w + f.
The quadratic equation x2 + mx + n = 0 has roots which are twice those of x2 + px + m = 0 and

Let x =

m, n and p 0. Find the value of

n
.
p

x
y
+
= 1 intersects the x and y axes at M and N respectively. If the coordinates of the point P lying
6
8
inside the triangle OMN (where 'O' is origin) are (a, b) such that the areas of the triangle POM, PON and PMN
are equal. Find
(i)
the coordinates of the point P and
(ii)
the radius of the circle escirbed opposite to the angle N.

8.

Line

9.

Starting at the origin, a beam of light his a mirror (in the form of a line) at the point A(4, 8) and is reflected at
the point B(8, 12). Compute the slope of the mirror.

10.

Find the solution set of inequality logx + 3 (x2 x) < 1.

11.

If the first 3 consecutive terms of a geometrical progression are the roots of the equation
2x3 19x2 + 57x 54 = 0 find the sum of infinite number of terms of G.P.

12.

Find the equation to the straight lines joining the origin to the points of intersection of the straight line

13.

Let f(x) = |x 2| + |x 4| |2x 6| . Find the sum of the largest and smallest values of f(x) if x [2, 8].

x
y
+
= 1 and the circle 5(x2 + y2 + bx + ay) = 9ab. Also find the linear relation between a and b so that
a
b
these straight lines may be at right angle.

Subject : Mathematics
1.

If

Extra Edge Assignment

x 1

x2

xa

x3

x4

xc

x2

x3

xb

Assignment 56

= 0, then all lines represented by ax + by + c = 0 pass through a firxed point. Find

the coordinates of that fixed point.


2.

If S1, S2, S3, ......, Sn, ....... are the sums of infinite geometric series whose first terms are 1, 2, 3, ....n, .....
and whose common ratio are

1 1 1
1
, , , ......
, .... respectively, then find the value of
2 3 4
n 1

2n 1
r 1

2
r

5
20
A
B
=
and tan
=
, then find the value of tan C.
6
37
2
2

3.

In any triangle if tan

4.

The radii r1, r2, r3 of escirbed circles of a traingel ABC are in harmonic progression. If its area is 24 sq. cm and
its perimeter is 24cm, find the length of its sides.

5.

Find the equation of a circle passing through the origin if the line pair, xy 3x + 2y 6 = 0 is orthogonal to
it. If this circle is orthogonal to the circle x2 + y2 kx + 2ky 8 = 0 then find the value of k.

6.

Find the locus of the centres of the circles which bisects the circumferences of the circles x 2 + y2 = 4 and
x2 + y2 2x + 6y + 1 = 0

7.

Find the equation of the circle whose radius is 3 and which touches the circle x 2 + y2 4x 6y 12 = 0
internally at the point (1, 1).

8.

Find the equation of the line such that its distance from the lines 3x 2y 6 = 0 and 6x 4y 3 = 0 is equal

9.

Find the range of the variable x satisfying the quadratic equation, x 2 + (2 cos ) x sin2 = 0 R.

10.

y
x
sin x(3 sin 2 x )
If tan = tan3 , then prove that sin y =
4 2
4 2
1 3 sin 2 x

11.

Number of natural between 100 and 1000 such that at least one of their digits is 7, is
(A) 225
(B) 243
(C) 252
(D) none

12.

The number of ways in which 100 persons may be seated at 2 round tables T 1 and T2, 50 persons being
seated at each is
(A)

99 !
25

(B)

100 !
50

(C)

100 !
2

(D) 100C50

13.

there are six periods in each working day of a school. Number of ways in which 5 subjects can be arranged
if each subject is allotted at least one period and no period remains vacant is
(A) 210
(B) 1800
(C) 360
(D) 120

14.

Consider 8 vertices of a regular octagon and its centre. If T denotes the number of triangles and S denotes
the number of straight lines that can be formed with these 9 points then T S has the value equal to
(A) 44
(B) 48
(C) 52
(D) 56

15.

A polygon has 170 diagonals. How many sides it will have ?


(A) 12
(B) 17
(C) 20

(D) 25

Extra Edge Assignment

Subject : Mathematics

Assignment 57

1.

4 normal distinguishable dice are rolled once. The number of possible outcomes in which atleast one die
shows up 2 is :
(A) 216
(B) 648
(C) 625
(D) 671

2.

There are counters available in x different colours. The counters are all alike except for the colour. The total
number of arrangements consisting of y counters, assuming sufficients number of counters of each colour,,
if no arrangement consists of all counters of the same colour is
(A) xy x
(B) xy y
(C) yx x
(D) yx y

3.

In a plane a set of 8 parallel lines intersects a set of n parallel lines, that goes in another direction, forming a
total of 1260 parallelograms. The value of n is
(A) 6
(B) 8
(C) 10
(D) 12

4.

A team of 8 students goes on an excursion, in two cars, of which on can seat 5 and the other only 4. If
internal arrangement inside the car does not matter then the number of ways in which they can travel, is
(A) 91
(B) 126
(C) 182
(D) 3920

5.

In a conference 10 speakers are present. If S1 wants to speak before S2 and S2 wants to speak after S3, then
the number of ways all the 10 speakers can give their speeches with the above restriction if the remaining
seven speakers have no objection to speak at any number is

6.

7.

10!
3
There are 8 different consonants and 6 different vowels. Number of different words of 7 letters which can be
formed, if they are to contain 4 consonants and 3 vowels if the three vowels are to occupy even places is
(A) 8P4 . 6P3
(B) 8P4 . 6C3
(C) 8P4 . 7P3
(D) 6P3 . 7C3. 8P4

(A) 10C3

n 1

r 0

(A )

n
2

(B) 10P8

Cr

Cr nCr 1

(C) 10P3

(D)

is equal to
(B)

n(n 1)
2(n 1)

(C)

n 1
2

(D)

n(n 1)
2

8.

How many words can be made with the letters of the words " GENIUS" if each word neither begins with G nor
ends in S is :
(A) 24
(B) 240
(C) 480
(D) 504

9.

Number of numbers greater than 1000 which can be formed using only the digits 1, 1, 2, 3, 4, 0 taken four at
a time is
(A) 332
(B) 159
(C) 123
(D) 112

10.

Identify the correct statements


(A)
Number of naughts standing at the end of (125)! is 30.
(B)
A telegraph has 10 arems and each are is capable of 9 distinct positions excluding the position of
rest. The number of signals that can be transmitted is 1010 1.
(C)
In a table tennis tournament, every player plays with every other player. If the number of games
played is 5050 then the number of players in the tournament is 100.
(D)
Number of numbers greater than 4 lacs which can be formed by using only the digits 0, 2, 2, 4, 4 and
5 is 90.

11.
12.

C6 + nC4 > n + 2C5 nC5 for all 'n' greater than :


(A) 8
(B) 9
n+1

(C) 10

(D) 11

The continued product, 2.6.10.14 ....... to n factors is equal to


(A) 2nPn
(B) 2nCn
(C) (n + 1)(n + 2)(n + 3) ......(n + n)
(D) 2n . (1.3.5 ....... 2n 1)

Extra Edge Assignment

Subject : Mathematics

1.

Assignment 58

The number of ways in which five different books to be distributed among 3 persons so that each person gets
at least one book, is equal to the number of ways in which
(A)
5 persons are allotted 3 different residential flats so that and each person is alloted at most one flat
and no two persons are alloted the same flat.
(B)
Number of parallelograms (some of which may be overlapping) formed by one set of 6 parallel lines
and other set of 5 parallel lines that goes in other direction.
(C)
5 different toys are to be distributed among 3 children, so that each child gets at least one toy.
(D)
3 mathematics professors are assigned five different lecturers to be delivered, so that each professor
gets at least one lecturer.

Comprehension (2 to 4)
Consider the circle S : x2 + y2 4x 1 = 0 and the line L : y = 3x 1. If the line L cuts the circle at A and B
then
2.

Length of the chord AB equal

3.

The angle subtended by the chord AB in the minor arc of S is

(A) 2 5

(A)
4.

7.

(B)

(B)

(D)

(C)

(D)

(C)

(B) 2 +

1
1
, 2 +

The solutions of the equation, (1 + cos x)


equation

(D) 10

tan

(B) cosx = 0

(C) +

1
1
,

In ABC if B =
(A) r = 5/2

(B) ( , 1)

, s a = 3 ; s c = 2, then
2
(B) = 12

(D) +

1
1
,
2
2

x
2 + sin x = 2 cosx are identical to the solutions of the
2
(C) sin 2x = 0

The solution of the equation log


(3 2x) < logcos x 2 (2x 1) is
cos x 2
(A) (1/2, 1)

10.

2
3

If and are the roots of a(x2 1) + 2bx = 0, then which one of the following are the roots of the same
equation ?

(A) sinx = 1
9.

(C)

How many baseball nines can be chosen from 13 candidates if A, B, C, D are the only candidates for two
positions and can play at no other position ?

(A) + ,
8.

5
6

(D) 10

If the equation of the circle on AB as diamter is of the form x 2 + y2 + ax + by + c = 0 then the magnitude of

the vector V a i bj ck has the value equal to


(A)

6.

(B)

(C) 5 2

Acute angle between the line L and the circle S is


(A)

5.

3
4

(B)

(D) sec (x/2) =

(C) (1/2, 3)

(D) (1, )

(C) r1 = 2

(D) R = 3

2n , n N

Extra Edge Assignment

Subject : Mathematics

Assignment 59

1.

The ends of a quadrant of a circle have the coordinates (1, 3) and (3, 1), then the centre of the such a circle
is
(A*) (1, 1)
(B) (2, 2)
(C) (2, 6)
(D) (4, 4)

2.

Centroid of the triangle the equations of whose sides are 12x 2 20xy + 7y2 = 0 and 2x 3y + 4 = 0 is
(A) (3, 3)
(B*) (8/3, 8/3)
(C) (3, 8/3)
(D) (8/3, 3)

3.

Through a point A on the x-axis a straight line is drawn parallel to y-axis so as to meet the pair of straight
lines ax2 + 2hxy + by2 = 0 in B and C. If AB = BC then
(A) h2 = 4ab
(B*) 8h2 = 9ab
(C) 9h2 = 8ab
(D) 4h2 = ab

4.

A, B and C are points in the xy plane such that A(1, 2) ; B(5, 6) and AC = 3BC. Then
(A) ABC is a unique triangle
(B) There can be only two such triangles
(C) No such triangle is possible
(D*) There can be infinite number of such triangle

5.

If A(1, p2) ; B(0, 1) and C(p, 0) are the coordinates of three points then the value of p for which the area of the
triangle ABC is minimum , is
(A)

(B)

(C)

or

(D*) none

6.

m, n are integer with 0 < n < m. A is the point (m, n) on the cartesian plane. B is the reflection of A in the line
y = x. C is the reflection of B in the y-axis, D is the reflection of C in the x-axis and E is the reflection of D in
the y-axis. The area of the pentagon ABCDE is
(A) 2m(m + n)
(B*) m(m + 3n)
(C) m (2m + 3n)
(D) 2m (m + 3n)

7.

The angle between the two tangents from the origin to the circle (x 7) 2 + (y + 1)2 = 25 equals
(A)

(B)

(C*)

(D) none

8.

The equation of the pair of bisectors of the angles between two straight lines is
12x2 7xy 12y2 = 0. If the equation of one line is 2y x = 0 then the equation of the other line is :
(A*) 41x 38y = 0
(B) 11x + 2y = 0
(C) 38x + 41y = 0
(D) 11x 2y = 0

9.

Consider a quadratic equation in Z with parameters x and y as Z2 xZ + (x y)2 = 0


The parameters x and y are the co-ordinates of a variable point P w.r.t. an orthonormal co-ordinate system in
a plane. If the quadratic equation has equals roots then the locus of P is
(A) a circle
(B) a line pair through the origin of co-ordinate with slope 1/2 and 2/3
(C) a line pair through the origin of co-ordinate with slope 3/2 and 2
(D*) a line pair through the origin of co-ordinate with slope 3/2 and 1/2

10.

Vertices of a parallelogram ABCD are A(3, 1), B(13, 6), C(13, 21) and D(3, 16). If a line passing through the
origin divides the parallelogram into two congruent parts then the slope of the line is
(A)

11.

(B*)

11
8

(C)

25
8

(D)

13
8

The distance between the chords of contact of tangents to the circle ; x 2 + y2 + 2gx + 2fy + c = 0 from the
origin and the point (g, f) is :
(A)

12.

11
12

g f
2

(B)

g2 f 2 c
2

(C*)

g2 f 2 c
2 g2 f 2

(D)

g2 f 2 c

2 g2 f 2

The locus of the center of the circles such that the point (2, 3) is the mid point of the chord 5x + 2y = 16 is
(A*) 2x 5y + 11 = 0
(B) 2x + 5y 11 = 0
(C) 2x + 5y + 11 = 0
(D) none

Extra Edge Assignment

Subject : Mathematics

Assignment 60

1.

The values of a, for which one of the roots of the equation 2x2 2(2a + 1) x + a(a + 1) = 0 is greater than a and
the other is smaller, is
(A) (1/2, )
(B) (0, 1)
(C) ( , 3) (0, )
(D) (, 1) (0, )

2.

Find the least value of a for which (5x + 1 + 51 x) ;

3.

Consider the quadratic polynomial f(x) = x2 px + q where f(x) = 0 has prime roots. If p + q = 11 and
a = p2 + q2 then find the value of f(a) where a is an odd positive integer.

4.

The distance between the two parallel lines is 1 unit. A point 'A' is chosen to lie between the lines at a
distance 'd' from one of them. Triangle ABC is equilateral with B on one line and C on the other parallel line.
The length of the side of the equilateral triangle is

every x R.

(A)

2
3

d2 d 1

(B*) 2

d2 d 1
3

a1/ 3
and (25x + 25x) adn the successive terms of an A.P. for
2

(C) 2 d2 d 1

(D)

d2 d 1

5.

Given A(0, 0) and B(x, y) with x (0, 1) and y > 0. Let the slope of the line AB equals m 1. Point C lies on the
line x = 1 such that the slope of BC equals m 2 where 0 < m 2 < m1. If the area of the triangle ABC can be
expressed as (m1 m2) f(x), then the largest possible value of f(x) is
(A) 1
(B) 1/2
(C) 1/4
(D*) 1/8

6.

P lies on the line y = x and Q lies on y = 2x. The equation for the locus of the mid point of PQ, if |PQ| = 4, is
(A) 25x2 + 36xy + 13y2 = 4
(B*) 25x2 36xy + 13y2 = 4
2
2
(C) 25x 36xy 13y = 4
(D) 25x2 + 36xy 13y2 = 4

7.

If the vertices P and Q of a triangle PQR are given by (2, 5) and (4, 11) respectively, and the point R moves
along the line N : 9x + 7y + 4 = 0, then the locus of the centroid of the triangle PQR is a straight line parallel
to
(A) PQ
(B) QR
(C) RP
(D*) N

8.

The value of 'c' for which the set, {(x, y) | x2 + y2 + 2x 1} {(x, y) | x y + c 0} contains only one point in
common is :
(A) ( , 1] [3, )
(B) {1, 3}
(C) {3}
(D*) {1}

9.

P is a point (a, b) in the first quadrant. If the two circles which pass through P and touch both the co-ordinate
axes angles, then :
(A) a2 6ab + b2 = 0
(B) a2 + 2ab b2 = 0
2
2
(C*) a 4ab + b = 0
(D) a2 8ab + b2 = 0

10.

In a triangle ABC, if A(2, 1) and 7x + 10y + 1 = 0 and 3x 2y + 5 = 0 are equations of an altitude and an
angle bisector respectively drawn from B, then equation of BC is
(A) x + y + 1 = 0
(B*) 5x + y + 17 = 0
(C) 4x + 9y + 30 = 0
(D) x 5y 7 = 0

11.

A tangent at a point on the circle x2 + y2 = a2 intersects a concentric circle C at two points P and Q. the
tangents to the circle C at P and Q meet at a point on the circle x 2 + y2 = b2 then the equation of circle 'C' is
(A*) x2 + y2 = ab
(B) x2 + y2 = (a b)2
2
2
2
(C) x + y = (a + b)
(D) x2 + y2 = a2 + b2

Extra Edge Assignment

Subject : Mathematics
1.

The angle at which the circles (x 1)2 + y2 = 10 and x2 + (y 2)2 = 5 intersect is


(A)

2.

Assignment 61

(B*)

(C)

(D)

AB is the diameter of a semicircle k, C is an arbitrary point on the semicircle (other than A or B) and S is the
centre of the circle inscribed into triangle ABC, then measure of
C

k
S

(A) angle ASB changes as C moves on k.


(B) angle ASB is the same for all positions of C but it cannot be determined without knowning the radius.
(C*) angle ASB = 135 for all C.
(D) angle ASB = 150 for all C.
3.

Tangents are drawn the circle x2 + y2 = 1 at the points where it is met by the circles,
x2 + y2 ( + 6) x + (8 2) y 3 = 0. being the variable. The locus of the point of intersection of these
tangents is
(A*) 2x y + 10 = 0
(B) x + 2y 10 = 0
(C) x 2y + 10 = 0
(D) 2x + y 10 = 0

4.

Triangle formed by the lines x + y = 0, x y = 0 and x + my = 1. If and m vary subject to the condition
2 + m2 = 1 then the locus of its circumcentre is
(A*) (x2 y2) = x2 + y2 (B) (x2 + y2)2 = (x2 + y2) (C) (x2 + y2) = 4x2 y2
(D) (x2 y2)2 = (x2 + y2)2

5.

ABCD is a square of unit area. A circle is tangent to two sides of ABCD and passes through exactly one of
its vertices. The radius of the circle is
(A*) 2 2

(B)

2 1

(C) 1/2

(D)

6.

A parallelogram has 3 of its vertices as (1, 2), (3, 8) and (4, 1). The sum of all possible x-coordinates for the
4th vertex is
(A) 11
(B*) 8
(C) 7
(D) 6

7.

The image of the pair of lines represented by ax2 + 2hxy + by2 = 0 by the mirror y = 0 is
(A) ax2 2hxy by2 = 0
(B) bx2 2hxy + ay2 = 0
2
2
(C) bx 2h xy + ay = 0
(D*) ax2 2h xy + by2 = 0

8.

Two circles are drawn through the points (1, 0) and (2, 1) to touch the axis of y. They intersect at an angle:

9.

4
3

(B) cos1
(C)
(D) tan1 1
5
4
2
A circle of constant radius 'a' passes through origin 'O' and cuts the axes of co-ordinates in points P and Q,
then the equation of the locus of the foot of perpendicular from O to PQ is

(A*) cot1

1
1
(A) (x2 + y2) 2 = 4a2
y
x

1
1
(C*) (x2 + y2)2 2 = 4a2
y
x
10.

1
1
(B) (x2 + y2)2 2 = a2
y
x
1
1
(D) (x2 + y2) 2 = a2
y
x

If a circle of constant radius 3k passes through the origin 'O' and meets co-ordinate axes at A and B
(A*) x2 + y2 = (2k)2
(B) x2 + y2 = (3k)2
(C) x2 + y2 = (4k)2
(D )x2 + y2 = (6k)2

Subject : Mathematics

1.

2.

Extra Edge Assignment

Assignment 62

Chords of the curve 4x + y x + 4y = 0 which subtend a right angle at the origin pass through a fixed point
whose co-ordinates are :
2

4
1 4
1 4
1 4
1
(A*) ,
(B) ,
(C) ,
(D) ,
5
5 5
5 5
5 5
5
2
2
Let x and y be the real numbers satisfying the equation x 4x + y + 3 = 0. If the maximum and minimum
values of x2 + y2 are M and m respectively, then the numerical value of M m is
(A) 2
(B*) 8
(C) 15
(D) none of these

3.

If the straight lines joining the origin and the points of intersection of the curve
5x2 + 12xy 6y2 + 4x 2y + 3 = 0 and x + ky 1 = 0
are equally inclined to the co-ordinate axes then the value of k :
(A) is equal to 1
(B*) is equal to 1
(C) is equal to 2
(D) does note exist in the set of real number

4.

A line meets the co-ordinate axes in A and B. A circle is circumscribed about the triangle OAB. If d1 and d2 are
the distances of the tangent to the circle at the origin O from the points A and B respectively, the diameter of
the circle is
(A)

2d1 d 2
2

(B)

d1 2d 2
2

(C*) d1 + d2

d1d2
(D) d d
1
2

5.

If the line y = mx bisects the angle between the lines ax 2 + 2h xy + by2 = 0 then m is a root of the quadratic
equation
(A*) hx2 + (a b) x h = 0
(B) x2 + h (a b) x 1 = 0
2
(C) (a b) x + hx (a b) = 0
(D) (a b) x2 hx (a b) = 0

6.

Tangents are drawn from any point on the circle x2 + y2 = R2 to the circle x2 + y2 = r2. If the joining the points
of intersection of these tangents with the first circle also touch the second, then R equals

7.

(A)

2r

(C)

2r

(D)

4r

2 3
3 5
2
A variable circle C has the equation x + y 2(t 3t + 1) x 2 (t 2t) y + t = 0, where t is a parameter
If the power of point P(a, b) w.r.t. the circle C is constant then the ordered pair (a, b) is
2

1
1

(A) ,
10 10

8.

(B*) 2r

1 1
,

(B*)
10 10

1 1

(C) ,
10 10

1
1
,

(D)
10
10

Let C be a circle with two diameters intersecting at an angle of 30 degrees. A circle S is tangent to both the
diameters and to C, and has radius unity. The largest radius of C is
(A*) 1 6 2

(B) 1 6 2

(C)

6 2 1

(D) none of these

9.

A variable circle C has the equation x2 + y2 2(t2 3t + 1) x 2(t2 + 2t) y + t = 0, where t is a parameter
The locus of the centre of the circle is
(A*) a parabola
(B) an ellipse
(C) a hyperbola
(D) pair of straight lines

10.

Let a and b represent the length of a right triangle's legs. If d is the diameter of a circle inscribed into the
triangle, and D is the diameter of a circle superscribed on the triangle, then d + D equals
y

(0, a) A
O

(A*) a + b

(B) 2(a + b)

B(b, 0)

(C)

1
(a + b)
2

(D)

a2 b2

Extra Edge Assignment

Subject : Mathematics

Assignment 63

MCQ
1.

Three vertices of a triangle are A(4, 3) ; B(1, 1) and C(7, k). Values of k for which centroid, orthocentre,
incentre and circumcentre of the ABC lie on the same straight line is/are :
(A) 7
(B*) 1
(C*) 19/8
(D) none

2.

The ara of triangle ABC is 20 cm2. The co-ordiantes of vertex A are (5, 0) and B are (3, 0) . The vertex C lines
on the line x y = 2. The co-ordinate of C are
(A) (5, 3)
(B*) (3, 5)
(C) (5, 7)
(D*) (7, 5)

3.

Two vertices of the ABC are at the points A(1, 1) and B(4, 5) and the third vertex lines on the striaght line
y = 5(x 3). If the area of the is 19/2, then the possible co-ordiantes of the vertex C are :
(A*) (5, 10)
(B*) (3, 0)
(C) (2, 5)
(D) (5, 4)

4.

A circle passes through the points (1, 1), (0, 6) and (5, 5). The points on this circle , the tangent(s) at which
is/are parallel to the straight line joining the origin to its centre is/are :
(A) (1 , 5)
(B*) (5, 1)
(C) (5, 1)
(D*) (1, 5)

5.

Line

x
y
x
y
+
= 1 cuts the co-ordinate axes at A(a, 0) and B(0, b) and the line
+
= 1 at A( a, 0)
a
b
a
b
and B(0, b). If the points A, B, A, B are concyclic then the orthocentre of the triangle ABA is :

(A) (0, 0)
6.

bb

(D) 0 ,
a

2 44

(B*) ,
5 5

(C*) (6, 4)

(D) (3, 5)

If one vertex of an equilateral triangle of side 'a' lies at the origin and the other lies on the line x
then the co-ordinates of the third vertex are :
(A*) (0, a)

8.

aa

(C*) 0 ,
b

Point M moved along the circle (x 4)2 + (y 8)2 = 20. Then it broke away from it and moving along a tangent
to the circle, cuts the x-axis at the point (2, 0). The co-ordinates of the point on the circle at which the
moving point broke away can be :
3 46

(A) ,
5 5

7.

(B*) (0, b)

3a a

(B*) 2 , 2

(C*) (0, a)

3 ,y=0

3a a

(D*) 2 , 2

The circles x2 + y2 + 2x + 4y 20 = 0 and x2 + y2 + 6x 8y + 10 = 0


(A*) are such that the number of common tangents on them is 2
(B) are not orthogonal
(C*) are such that the length of their common tangent is 5(12/5) 1/4
(D*) are such that the length of their common chord is 5

3
.
2

9.

The centre(s) of the circle(s) passing through the points (0, 0), (1, 0) and touching the circle x 2 + y2 = 9 is/are

10.

The circles x2 + y2 2x 4y + 1 = 0 and x2 + y2 + 4x + 4y 1 = 0


(A) touch internally
(B*) touch externally
(C*) have 3x + 4y 1 = 0 as the common tangent at the point of contact.
(D) have 3x + 4y + 1 = 0 as the common tangent at the point of contact.

3 1
(A) ,
2 2

1 3
(B) ,
2 2

1 1/ 2
(C*) , 2
2

1
1/ 2
(D*) , 2
2

You might also like